You are on page 1of 42

Study Notes – Internal Medicine James Lamberg 01Feb2010

Textbooks: Cecil Essentials of Medicine, Hospital Medicine Secrets, First Aid for Medical Clerkship
Introductory Guide: Primer to the Internal Medicine Clerkship, 2nd Edition, by Picchioni

Common Problems in Internal Medicine


Cardiovascular: Acute Coronary Syndromes, Congestive Heart Failure, Valvular Heart Disease, Atrial Fibrillation
and Anticoagulation, Hypertension
Endocrine: Diabetes Mellitus, Hypothyroidism, Hyperthyroidism, Osteoporosis, Disorders of Calcium Metabolism
Hematology: Anemia, Coagulopathies
Gastro: Hepatitis, Peptic Ulcer Disease, Gastroesophageal Reflux Disease, Diarrhea and Constipation
Oncology: Hematological Malignancy, General Care of the Cancer Patient, Management of Pain
Nephrology: Electrolyte Disturbances, Acid-Base Disorders, Acute and Chronic Renal Failure
Rheumatology: Rheumatoid Arthritis, Osteoarthritis, Monoarthritides, Polyarthritides
Pulmonary: DVT and Pulmonary Embolism, Chronic Bronchitis and Asthma, Emphysema
Infectious: Fever of Unknown Origin, Acquired Immune Deficiency Syndrome, Pneumonia, Urinary Tract
Infection, Cellulitis, Subacute Bacterial Endocarditis
Allergy: Urticaria
Neurology: Cerebrovascular Disease, Headache, Dementia and Coma
Dermatology: Dermatological Manifestations of Chronic Medical Disease

Procedures: NEJM Videos In Clinical Medicine: http://content.nejm.org/misc/videos.dtl

How To Succeed In Clerkship – First Aid For The Medicine Clerkship (Stead, Stead, & Kaufman)
Be On Time: Team rounds usually begin between 7am and 8am. Give yourself at least 10 minutes per patient for
pre-rounding to learn about events that occurred overnight or lab/imaging results.
Dress In A Professional Manner: Regardless of what the attending wears. A short white coat should be worn over
your professional dress clothes unless it is discouraged (e.g. pediatrics).
Act In A Pleasant Manner: The medical rotation is often difficult, stressful, and tiring. Smooth out your experience
by being nice to be around. Smile a lot and learn everyone’s name. Don’t be afraid to ask how your resident’s
weekend was. If you do not under- stand or disagree with a treatment plan or diagnosis, do not “challenge.” In-
stead, say “I’m sorry, I don’t quite understand, could you please explain…” Show kindness and compassion toward
your patients. Never participate in callous talk about patients.
Take Responsibility: Know everything there is to know about your patients: their history, test results, details about
their medical problem, and prognosis. Keep your intern or resident informed of new developments that they might
not be aware of, and ask them for any updates you might not be aware of. Assist the team in developing a plan;
speak to radiology, consultants, and family. Never give bad news to patients or family members without the
assistance of your supervising resident or attending.
Respect Patient’s Rights:
1) All patients have the right to have their personal medical information kept private. This means do not discuss the
patient’s information with family members without that patient’s consent, and do not discuss any patient in
hallways, elevators, or cafeterias.
2) All patients have the right to refuse treatment. This means they can refuse treatment by a specific individual (you,
the medical student) or of a specific type (no nasogastric tube). Patients can even refuse life- saving treatment. The
only exceptions to this rule are if the patient is deemed to not have the capacity to make decisions or understand
situations, in which case a health care proxy should be sought, or if the patient is suicidal or homicidal.
3) All patients should be informed of the right to seek advanced directives on admission. Often, this is done by the
admissions staff, in a booklet. If your patient is chronically ill or has a life-threatening illness, address the subject of
advanced directives with the assistance of your attending.
More Tips: Volunteer, be a team player, be honest, and keep patient information handy.
Present In An Organized Manner: “This is a [age]-year-old [gender] with a history of [major history such as HTN,
DM, coronary artery disease, CA, etc.] who presented on [date] with [major symptoms, such as cough, fever, and
chills] and was found to have [working diagnosis]. [Tests done] showed [results]. Yesterday, the patient [state
important changes, new plan, new tests, new medications]. This morn- ing the patient feels [state the patient’s
words], and the physical exam is significant for [state major findings]. Plan is [state plan]."
Presenting A Chest Radiograph (CXR):
1) Technique: Rotation, anteroposterior (AP) or posteroanterior (PA), penetration, inspiratory effort.
2) Bony structures: Look for rib, clavicle, scapula, and sternum fractures.

DO NOT DISTRIBUTE -1-


Study Notes – Internal Medicine James Lamberg 01Feb2010

3) Airway: Look for tracheal deviation, pneumothorax, and pneumomediastinum.


4) Pleural space: Look for fluid collections, which can represent hemothorax, chylothorax, and pleural effusion.
5) Lung parenchyma: Look for infiltrates and consolidations: These can represent pneumonia, pulmonary
contusions, hematoma, or aspiration. The location of an infiltrate can provide a clue to the location of pneumonia:
* Obscured right (R) costophrenic angle = Right lower lobe
* Obscured left (L) costophrenic angle = Left lower lobe
* Obscured R heart border = Right middle lobe
* Obscured L heart border = Left upper lobe
6) Mediastinum: Look at size of mediastinum—a widened one (> 8 cm) goes with aortic dissection. Look for
enlarged cardiac silhouette (> 1⁄2 thoracic width at base of heart), which may represent congestive heart failure
(CHF), cardiomyopathy, or pericardial effusion.
7) Diaphragm: Look for free air under the right hemidiaphragm (suggests perforation). Look for stomach, bowel, or
nasogastric tube (NGT) above diaphragm (suggests diaphragmatic rupture).
8) Tubes and lines:
* Identify all tubes and lines.
* An endotracheal tube should be 2cm above the carina. Common mistake is right bronchus intubation.
* A chest tube (and proximal hole) should be in the pleural space (not in the lung parenchyma).
* An NGT should be in the stomach and uncoiled.
* The tip of a central venous catheter should be in the superior vena cava (not in the right atrium).
* The tip of a Swan–Ganz catheter should be in the pulmonary artery.
* The tip of a transvenous pacemaker should be in the right atrium.
Presenting A Chest Radiograph (Mnemonic Method):
Mnemonic: RRR, RIP, ABCDEFGH
* Right: patient, procedure, date
* Rotation: spinous processes are to line up vertically, equal space between clavicles
* Inspiration: should show 8 ribs
* Penetration: spinous processes should just be visible through the vertebrae
* Airway: carina and tracheal deviation
* Bones: look at clavicles, vertebrae, scapula, and ribs for fractures
* Cardiac silhouette: > 1/2 total chest width could be CHF, determine if edges are clear
* Diaphragm: elevated or depressed, right should be higher, no air under diaphragm
* Effusions: check borders and edges for fluid levels, hemothorax, atelectasis, pneumothorax
* Fields: infiltrates, masses, objects, size (large in emphysema, small in chronic bronchitis)
* Gadgets: ET tubes, central lines, chest tubes, pacemakers, ECG monitors, mention this after RRR RIP in ICU
* Hilum: any masses or disturbances
Presenting An Electrocardiogram (ECG):
1) Rate: The rate is [number of] beats per minute (bpm):
* The ECG paper is scored so that one big box is 0.20 seconds. These big boxes consist of five little boxes, each of
which is 0.04 seconds.
* A quick way to calculate rate when the rhythm is regular is the mantra: 300, 150, 100, 75, 60, 50 (= 300 / # large
boxes), which is measured as the number of large boxes between two QRS complexes. Therefore, a distance of one
large box between two adjacent QRS complexes would be a rate of 300, while a distance of five large boxes
between two adjacent QRS complexes would be a rate of 60.
* For irregular rhythms, count the number of complexes that occur in a 6-second interval (30 large boxes) and
multiply by 10 to get a rate in bpm.
2) Rhythm: The rhythm is [sinus]/[atrial fibrillation]/[atrial flutter] or other:
* If p waves are present in all leads and upright in leads I and aVF, then the rhythm is sinus. Lack of p waves
suggests a disorganized atrial rhythm, a junctional rhythm, or a ventricular rhythm. A ventricular rhythm (V Fib or
V Tach) is an unstable one (could spell imminent death), and you should be getting ready for advanced cardiac life
support (ACLS).
* Normal sinus rhythm is usually a regular narrow-complex rhythm with each QRS complex preceded by a p wave.
3) Axis: The axis is [normal]/[deviated to the right]/[deviated to the left]:
* If I and aVF are both upright or positive, then the axis is normal. * If I is upright and aVF is upside down, then
there is left axis deviation (LAD).
* If I is upside down and aVF is upright, then there is right axis deviation (RAD).
* If I and aVF are both upside down or negative, then there is extreme RAD.

DO NOT DISTRIBUTE -2-


Study Notes – Internal Medicine James Lamberg 01Feb2010

4) Intervals: The [PR]/[QRS] intervals are [normal]/[shortened]/[widened]:


* Normal PR interval = 0.12 to 0.20 seconds:
* Short PR is associated with Wolff–Parkinson–White syndrome (WPW).
* WPW syndrome is characterized by a “delta” wave, or slurred up-stroke of QRS complex.
* Long PR interval is associated with heart block of which there are three types:
* First-degree block: PR interval > 0.20 seconds (one big box)
* Second-degree (Mobitz type I or Wenckebach) block: PR interval lengthens progressively until a QRS is dropped.
* Second-degree (Mobitz type II) block: PR interval is constant, but one QRS is dropped at a fixed interval.
* Third-degree heart block: Complete AV dissociation Normal QRS interval ≤ 0.12 seconds:
* Prolonged QRS is seen when the beat is initiated in the ventricle rather than the sinoatrial node, when there is a
bundle branch block, and when the heart is artificially paced with longer QRS intervals. Prolonged QRS is also
noted in tricyclic overdose and Wolfe–Parkinson–White syndrome.
5) Wave morphology:
A. Ventricular hypertrophy: There [is/is no] [left/right] [ventricular/atrial] hypertrophy:
* There are multiple criteria for determining right (RVH) and left ventricular hypertrophy (LVH).
Clues for LVH:
* RI>15mm, RI,II or aVF >20mm, RaVL>11mm, RV5 or RV6 >26mm, RI +SIII >25mm, R+S in Vlead>45mm,
SV1 +RV5 or RV6 >35mm
Clues for RVH:
* RV1>7mm, SV1<2mm, R/S ratio inV1 >1, RAD of 110deg or more
B. Atrial hypertrophy:
* Right atrial hypertrophy: tall or peaked p waves in limb or precordial leads
* Left atrial hypertrophy: broad or notched p waves in limb leads
C. Ischemic changes: There [are/are no] S-T wave [depressions/elevations] or [flattened/inverted] T waves. Presence
of Q wave indicates an old infarct.
D. Bundle branch block: There [is/is no] [left/right] bundle branch block. Clues:
* Presence of RSR’ wave in leads V1-V3 with ST depression and T wave inversion goes with RBBB.
* Presence of notched R wave in leads I, aVL, and V4-V6 goes with LBBB.

Top 100 Secrets – Medical Secrets (4th, Zollo)


1) The treatment of severe sepsis syndrome should be based on efficient resuscitation, effective antimicrobial
therapy, elimination of secondary infections, euglycemia, early targeted and specific drug therapy, and establishment
of therapeutic goals.
2) Acute pulmonary embolism (PE) is a difficult diagnosis to establish despite newer advances in imaging;
approximately 50% of cases are diagnosed post mortem.
3) In the approach to suspected PE, keep in mind the prudent use of key diagnostic tests: (1) rapid d-dimer by
ELISA is an effective screening test; (2) chest CT can help detect most PEs; and (3) a negative Doppler venous
ultrasound of the legs does not exclude the diagnosis of PE.
4) The most common etiologic agent implicated in acute bacterial meningitis in the U.S. is Streptococcus
pneumoniae.
5) In the newly diagnosed HIV patient, in addition to routine adult immunizations, immunizations against
pneumococcal pneumonia, influenza, and both hepatitis A and B are indicated.
6) Metabolic syndrome is diagnosed on the basis of abdominal obesity, hypertriglyceridemia, low HDL cholesterol
levels, hypertension, and fasting hyperglycemia.
7) Pituitary tumors cause problems for patients by two main mechanisms: mass effect, which applies pressure to
surrounding structures, and endocrine hyperfunction, which results in excessive secretion of a particular anterior
pituitary hormone.
8) A key concept in evaluating patients with hyperfunctioning endocrine tumors is that biochemical diagnosis should
always precede anatomic localization.
9) The best initial screening test for evaluation of thyroid status is the TSH, since it is the most sensitive measure of
thyroid function in the majority of patients. The one exception is patients with pituitary/hypothalamic dysfunction,
in whom TSH cannot reliably to assess thyroid function.
10) The most common presentation of hypogonadism is erectile dysfunction and decreased libido in men and
amenorrhea and infertility in women.
11) All patients with coronary artery disease (CAD), CAD-equivalent diseases, or diabetes should be treated
aggressively to reach the LDL-cholesterol target of 100 mg/dL.

DO NOT DISTRIBUTE -3-


Study Notes – Internal Medicine James Lamberg 01Feb2010

12) Diabetics and patients with vascular disease should be treated with a statin lipid-lowering drug to prevent heart
disease and stroke, regardless of the blood low-density lipoprotein (LDL) cholesterol level, age (from 40 to 79
years), or gender.
13) The goal blood pressure is < 130/80 mmHg in hypertensive subjects with diabetes mellitus and/or chronic
kidney disease.
14) The single most life-saving treatment strategy in patients with acute ST-elevation myocardial infarction is to
rapidly achieve complete reperfusion of the infarct-related artery by mechanical (balloon angioplasty or stenting) or
pharmacologic means (thrombolysis).
15) Angiotensin-converting enzyme inhibitors (or angiotensin receptor blockers) and beta-adrenergic blockers are
effective in reducing cardiovascular complications and improving survival in patients with systolic heart failure and
are recommended in all patients with no contraindications to these drugs.
16) Noninvasive stress testing has the best predictive value for detecting CAD in patients with an intermediate (30-
80%) pretest likelihood of CAD and is of limited value in patients with very low (< 30%) or very high (> 80%)
likelihood of CAD.
17) In patients with Coccidioides immitis infections, higher titers of complement-fixing antibodies suggest more
extensive disease, and rising titers suggest worsening disease.
18) Patients who present with flaccid paralysis during the summer months should be evaluated for West Nile virus
infection.
19) A febrile patient with rash who presents to the emergency department during May to September in the South
Atlantic and West South Central states should receive empirical doxycycline therapy for suspected Rocky Mountain
spotted fever.
20) Community-acquired methicillin-resistant Staphylococcus aureus that is susceptible to clindamycin but resistant
to erythromycin should not be treated with clindamycin because of the possibility for induction of resistance.
21) In patients with disseminated candidiasis, IV catheters should be removed and ophthalmologic examinations
performed to evaluate for the presence of retinal disease.
22) Transmission of Borrelia burgdorferi (the causative agent of Lyme disease) from an infected Ixodes tick to a
susceptible human requires the tick to have fed on the human for at least 40 hours.
23) Porcelain gallbladder is an incidental finding, more common in women who have gallstones. Because up to 50%
of patients develop gallbladder carcinoma, prophylactic cholecystectomy is recommended.
24) Three liters of Coca-Cola administered via nasogastric lavage over a 12-hour period can dissolve gastric
bezoars. It is thought that the cola acidifies the gastric contents and liberates carbon dioxide in the stomach, resulting
in the disintegration of phytobezoars.
25) Regardless of what is done, GI bleeding stops spontaneously in about 80% of patients.
26) Patients with hereditary nonpolyposis colorectal cancer syndrome have a higher-than-average risk of developing
colon and gastric cancer.
27) About 90% of patients with primary sclerosing cholangitis have underlying ulcerative colitis, but less than 10%
of all patients with ulcerative colitis have primary sclerosing cholangitis.
28) In patients with suspected perforation, the minimum amount of free air that can be detected on an upright chest
x-ray is 12mL.
29) The three major openings in the diaphragm through which hernias may occur are the esophageal hiatus (most
common), foramen of Bochdalex (3-5%, usually left-sided), and foramen of Morgagni (rare).
30) In a patient who has a malignancy involving the right hilum, look at the hand veins. If the veins in the hands are
distended and do not collapse when the arms are lifted over the head, there is a high chance of superior vena cava
obstruction.
31) In high-risk patients, the chance of developing breast cancer can be reduced by about 50% with the use of
tamoxifen.
32) If a patient with lung cancer presents with hoarseness, look for vocal cord paralysis, a sign of mediastinal
involvement (recurrent laryngeal nerve) that renders the patient inoperable.
33) Patients with head and neck cancer have a 30% chance of developing another cancer somewhere in the
aerodigestive tract (head and neck, lung, or esophagus), especially if they continue to smoke and drink.
34) If a patient presents with hypercalcemia, look for a squamous cell cancer (lung, esophagus, head and neck,
cervix, anus).
35) Up to 15% of breast cancers may not be detectable by mammogram. If the patient has a clinically suspicious
lump, perform a biopsy.
36) The presence of bilateral small kidneys in a patient with azotemia confirms chronic renal failure.

DO NOT DISTRIBUTE -4-


Study Notes – Internal Medicine James Lamberg 01Feb2010

37) In a diabetic patient with proteinuria, the presence of concomitant retinal disease suggests strongly (90%
correlation) that the renal manifestations are due to diabetes.
38) Treatment of anemia of chronic renal failure by recombinant human erythropoietin is highly effective, but
correction of iron deficiency and iron supplementation by oral or intravenous route is simpler, cheaper, and often by
itself effective therapy.
39) In resistant hypertension, especially in younger (< 20 yr) or older (> 70 yr) patients, consider and rule out
renovascular hypertension.
40) New onset of nephrotic proteinuria in an elderly patient warrants exclusion of an underlying malignancy.
41) The principal mechanism of bicarbonate reabsorption in the proximal tubule is through Na+-H+ exchanger
(NHE3) activity.
42) D-lactic acidosis is characterized by increased serum anion gap, metabolic acidosis, and episodic
encephalopathy in patients with short bowel syndrome.
43) Ethylne glycol (antifreeze) toxicity is characterized by high anion gap metabolic acidosis, neurotoxicity in the
form of ataxia, seizures, and calcium oxalate crystals in the urine.
44) Bartter's syndrome is a disorder associated with normotensive hyperaldosteronism, secondary to juxtaglomerular
hyperplasia, hypokalemic metabolic alkalosis, and severe renal potassium wasting.
45) Hyperkalemia is an important side effect of both ACE inhibitors and ARBs, but the problem is less frequent and
smaller in magnitude with ARBs because of their less pronounced effects on aldosterone levels.
46) Hypochromic microcytic anemias are the most encountered anemias in hospitalized and ambulatory patients.
47) Both iron-deficiency anemia and anemia of chronic disease have a low transferrin saturation. In iron-deficiency
anemia, the TIBC is often increased, whereas anemia of chronic disease is marked by an unusually low TIBC.
48) The main clinical manifestations of sickle hemoglobinopathies are hemolytic anemia, chronic end-organ
damage, periodic vaso-occlusive disease ("crises"), and hyposplenism.
49) The triad of thrombocytopenia, fragmentation hemolysis, and fluctuating neurologic signs suggests thrombotic
thrombocytopenic purpura (TTP), perhaps the most spectacular of the fragmentation syndromes.
50) The cytogenetic marker of chronic myelogenous leukemia is the 9:22 translocation, in which portions of the
long arms of chromosomes 9 and 22 are exchanged, resulting in a shortened 22 or Philadelphia chromosome (Ph1).
Some patients with acute lymphoblastic leukemia (ALL) also have 9:22 translocations (poor prognostic marker).
51) The classic cell seen in the lymph nodes of patients with Hodgkin's disease is the Reed-Sternberg (RS) cell, a
large cell with two nuclei, each possessing a distinct nucleolus.
52) Secondary monoclonal gammopathy must be distinguished from the monoclonal gammopathy associated with
multiple myeloma, benign monoclonal gammopathy of uncertain significance, solitary plasmacytoma, amyloidosis,
lymphoma, and Waldenström's macroglobulinemia.
53) Deep venous thrombosis in a young person, a family history of thrombosis, thrombosis at unusual sites (such as
the mesenteric vein), or recurrent thrombosis without precipitating factors suggests a hypercoagulable state.
54) Any condition that leads to V/Q mismatching can cause hypoxemia. Most pulmonary disorders are associated
with some degree of V/Q mismatching. This is the most common cause of hypoxemia and is responsive to oxygen
therapy.
55) Assuming that you are at sea level and breathing room air, an easy way to calculate the A-a difference is as
follows: (150-40/0.8) - PaO2 measured by ABG.
56) Although the anterior segment of the upper lobes may be affected by TB, a lesion found only in the anterior
segment suggests a diagnosis other than TB (e.g., malignancy).
57) Incidence of lung cancer now exceeds breast cancer in women. Women develop lung cancer at an earlier age
and after fewer years of smoking.
58) Pleural fluid glucose < 30 mg/dL and pH < 7.30 suggest rheumatoid effusion, TB, lupus, or malignancy.
59) Mesothelioma, a pleural malignancy associated with asbestosis exposure, is not associated with tobacco use.
60) Early, aggressive intervention with disease-modifying antirheumatic drugs reduces the morbidity (deformity
leading to reduced functionality and disability) and mortality associated with rheumatoid arthritis.
61) Antinuclear antibody (ANA) titers are not associated with activity of disease.
62) COX2 NSAIDs are no more efficacious than older standard NSAIDs but are significantly less toxic.
63) A patient with low positive rheumatoid factor (RF) and arthralgia should be checked for hepatitis C, which can
produce a low-grade synovitis and cryoglobulins (which in turn can produce a falsely positive RF).
64) Always check for Sjögren's antibodies (SSA/SSB) and phospholipid antibodies in a young woman with lupus
before conception. Sjögren's antibodies increase the risk of neonatal lupus (rash, thrombocytopenia, heart block),
and phospholipid antibodies can significantly increase the risk for miscarriage, premature labor, or intrauterine
growth delay.

DO NOT DISTRIBUTE -5-


Study Notes – Internal Medicine James Lamberg 01Feb2010

65) Packed red cells in freshly acquired blood may include lymphocytes that can mount a graft-versus-host reaction
if the patient's own immune system is unable to rapidly kill and inactivate these transfused allogeneic leukocytes.
66) Intranasal steroids are the single most effective drug for treatment of allergic rhinitis. Decongestion with topical
adrenergic agents may be needed initially to allow corticosteroids access to the deeper nasal mucosa.
67) The clinical manifestations of anaphylaxis include flushing, sense of foreboding, urticaria or angioedema,
pruritus, hoarseness, stridor, bronchospasm, hypotension, tachycardia, nausea, vomiting, abdominal pain, diarrhea,
headache, and syncope.
68) ACE inhibitors are often-forgotten causes of angioedema and chronic cough.
69) Chronic urticaria may require treatment with a combination of both H1 and H2 antihistamines, reflecting the
distribution of these receptors in the skin. Work-up for an allergic etiology is rarely informative.
70) Beta blockers should be avoided whenever possible in patients with asthma because they may accentuate the
severity of anaphylaxis, prolong its cardiovascular and pulmonary manifestations, and greatly decrease the
effectiveness of epinephrine and albuterol in reversing the life-threatening manifestations of anaphylaxis.
71) HIV infection is preventable and treatable but never curable.
72) If you are thinking of mononucleosis as a diagnosis, think about and test for HIV.
73) Adherence to anti-HIV therapy must be > 95% for a durable response. HIV treatment guidelines change
frequently - always verify your information.
74) A person under care for HIV should not develop pneumocyotic pneumonia (PCP). It is entirely preventable.
75) There is a critical interaction between HIV and tuberculosis. If one infection is present, look for the other.
76) If you have diagnosed one sexually transmitted disease (STD), you must consider others, especially HIV.
77) Most back pain is not caused by a radiculopathy.
78) The most common cause of dizziness is benign paroxysmal positional vertigo.
79) The leading causes of death after a stroke are medical complications, not the stroke itself.
80) Heparin has no value in the acute treatment of strokes.
81) The sudden onset of a severe headache may indicate an intracranial hemorrhage.
82) Coma is usually caused by medical problems, not neurologic ones.
83) Elective surgery should be postponed for further evaluation if the patient has signs or symptoms of unstable or
inadequately treated chronic disease.
84) Patients who have undergone coronary revascularization within 5 years of a proposed elective surgery and have
no signs or symptoms of recurrent ischemia can usually undergo surgery without further evaluation.
85) Acute dyspnea in a patient who has had major surgery should raise the suspicion of pulmonary embolism, even
if the patient has received prophylaxis.
86) All patients who take oral agents for diabetes may continue them until the day of surgery unless they have
chronic liver or renal disease or are on a first-generation sulfonylurea. In these cases the oral agent should be held at
least several days in advance of the surgery.
87) Pacemakers and implanted cardioverters/defibrillators should be assessed both before and after surgery,
radiation therapy, or lithotripsy.
88) Surgery patients on any antiplatelet agent should be told when to stop the medication before surgery and when to
resume it afterward to minimize perioperative bleeding.
89) Strict bed rest is not needed for the treatment of acute lumbosacral strain.
90) Influenza virus vaccination reduces hospitalization and death from influenza and its complications in elderly and
high-risk patients.
91) Always examine the feet and pedal pulses of diabetic patients regularly, looking for ulcerations, injury, or
reduced blood flow.
92) Closely monitor patients with blood pressure measurements defined as "prehypertension," and encourage
lifestyle changes to prevent progression to hypertension.
93) Reduce the risk of hip fracture in elderly and high-risk patients with calcium and vitamin D supplements,
exercise prescription, hip pads, and medications to treat osteoporosis, when indicated.
94) Assess a woman's risk of coronary disease, stroke, thromboembolism, and breast cancer before prescribing
estrogen/progesterone therapy in menopause.
95) Older adults currently constitute the fastest-growing population in the United States - a trend that is expected to
continue for the foreseeable future.
96) Commonly used instruments for a comprehensive geriatric assessment include the Mini Mental State Exam, the
Geriatric Depression Scale, activities of daily living, instrumental activities of daily living, and assessment of
stability and mobility (e.g., Tinnetti or "Get Up and Go" test).
97) Dementia and short-term memory loss are not caused by aging.

DO NOT DISTRIBUTE -6-


Study Notes – Internal Medicine James Lamberg 01Feb2010

98) Delirium carries tremendous mortality and morbidity rates and should be identified, worked up aggressively, and
treated as any medical emergency.
99) Diastolic dysfunction, as distinct from systolic dysfunction, results from impaired relaxation in heart failure with
preserved ejection fraction and may account for half of all cases of heart failure in people over 80. Although the
symptoms of diastolic and systolic dysfunction may be similar, the traditional therapy for systolic dysfunction can
actually worsen ventricular filling and increase the risk of orthostasis and syncope in cases of diastolic dysfunction.
100) Fifteen percent of elderly patients who fall and fracture a hip report prior falls. It is essential to ask about falls,
assess for fall risk, and then act accordingly, given the significant mortality and morbidity of hip fractures.

Kaplan Videos – Neurology with Dr. Jacob Levy, MD


Even if you do not know the answer to a question, try to think what the answer could be. Always ask what is the
most likely diagnosis, why is it the most likely diagnosis, what is the next step in management (therapy vs.
diagnostics), what is the first diagnostic test to order, what is the best or most accurate diagnostic test to order?
Spinal Cord Compression
* A 61yo AAM is brought to the ED complaining of back pain that started gradually three days ago. He describes
the pain as band-like around the abdomen without radiation. His past medical history is significant for prostate
cancer diagnosed three years ago. First thing to think about is if this is an emergency or not an emergency; meaning
do we need to intervene right now or can we treat with something like analgesics and follow-up.
* The prostate cancer is important because metastatic disease to the spine could be compressing the spinal cord.
Other worrisome cancers could be breast, lung, multiple myeloma, lymphoma. Other worrisome signs with back
pain would be fever, urinary incontinence, urinary retention, fecal incontinence, sexual dysfunction in males,
bilateral lower extremity weakness. So just the history of cancer with back pain means we should be evaluating this
acutely. If we’re suspecting spinal cord compression, we should see upper motor neuron lesion signs below the level
of the compression. Signs would include hyperreflexia, increased tone, positive Babinski sign, spastic paralysis.
* Patients with spinal cord compression who are unable to lift up their limbs against gravity at the time of
compression have a 5% or less chance of being able to ambulate after their episode of compression. If the
compression is caught early enough when the patient is able to ambulate, their ability to ambulate after proper
treatment is about 80%.
* Most likely diagnosis is spinal cord compression. Next step in the management of this patient is give
dexamethasone, not MRI of the spine, not x-ray, not bone scan. Consider therapy before diagnostics in management
when the patient has an emergency. Best initial test is a spinal x-ray, sensitive maybe 80% of the time. Most
accurate test is MRI of the spine.
* Radiation therapy and surgical decompression are generally left for after the diagnosis is made. Say you gave
dexamethasone and diagnosed, now there is an abscess or hematoma compressing the spinal cord, then do surgical
decompression. Say it is lymphoma or cancer compressing the spinal cord, then do radiation.
Syringomyelia, Vitamin B12 Deficiency & Anterior Spinal Artery Infarction
* A 25yo man comes to the Emergency Department status post motor vehicle accident. The ED physician has
addressed the airway, breathing, circulation, and calls you to evaluate the patient. You perform a thorough
neurological exam, finding (motor) lower extremity weakness 4/5 bilaterally with some hyperreflexia, (CN) cranial
nerves intact, (sensory) pain/temperature lost on lower extremities, vibration/position intact, and light touch intact.
* Most likely diagnosis is syringomyelia, with a pool of fluid developing in the spinal cord. Pain and temperature
lost because these spinal cord tracts are centrally located (spinothalamic), while vibration/position and light touch
are in posterior column.
* Best initial test for syringomyelia is an MRI. Most sensitive test is an MRI. Treatment is often surgical. If there is
an anatomic problem in the spinal cord, there should be an anatomic solution.
* Vitamin B12 deficiency causes subacute combined degeneration of the cord. Characteristic signs of vitamin B12
deficiency are loss or proprioception and vibration with intact pain/temperature, usually with ataxia and pyramidal
signs like hyperreflexia and positive Babinski.
* Anterior spinal artery feeds the sensory neurons involved in pain and temperature. So an anterior spinal artery
infarct would have lost pain/temperature sense, sudden onset of flaccid paresis, and the presence of intact
proprioception/vibration sense.
* Patient presents with signs of cord compression. An MRI shows osteomyelitis compressing the anterior cord. Best
test is MRI, first initial step is give dexamethasone. Best initial test is x-ray, even if they’re showing you MRI.
Cerebrovascular Accidents (CVA)
* 56yo woman is brought to the ED by her daughter, complaining of sudden onset of right upper extremity weakness
that started while she was watching television in the morning. Her daughter became concerned when her mother was

DO NOT DISTRIBUTE -7-


Study Notes – Internal Medicine James Lamberg 01Feb2010

unable to talk in response to her questions. On neurologic exam, you note right upper extremity weakness with
pronator drift and right facial palsy. When you question the patient, she seems to understand what is being said but
she cannot clearly respond.
* Most likely diagnosis is cerebrovascular accident (CVA). Pathophysiology is lack of blood supply to section of the
brain. Could be an embolus coming from the atrium in a patient with atrial fibrillation. It could be a bleed in a
patient with hypertension. It could also be thrombosis, where the clot forms in the brain itself (not an embolus from
a distant site). Hypotension can also induce ischemia, such as in a watershed infarct.
* Focal neurologic deficit of sudden onset is very likely to be a stroke.
* Risk factors for cerebrovascular disease are the same as coronary artery disease, peripheral vascular disease, or
carotid artery disease. General atherosclerosis risk factors include smoking, high cholesterol, diabetes, obesity, male
gender, hypertension, increased age, HIV, and alcohol abuse.
* Important risk factors include atrial fibrillation, recent transient ischemic attack (TIA) with similar symptoms,
valvular disease (especially mitral stenosis).
* There is an anterior circulation and a posterior circulation to the brain. Anterior is the middle cerebral artery
(MCA) and anterior cerebral artery (ACA). Posterior is vertebral arteries forming the basilar artery and the posterior
cerebral artery (PCA).
* Patients with MCA infarcts present with contralateral hemiparesis involving the face and the arm more extensively
than the lower extremity. So, patient would have facial droop and weak upper extremity.
* Patients with ACA infarcts presents with contralateral hemiparesis involving the leg more than the face/arm.
Mnemonic is “put your best foot forward” for foot/leg affected with forward/anterior artery. ACA ischemia may
involve urinary incontinence and personality changes (Phineas Gage).
* Homunculus, “little man on your brain,” has to sleep comfortably at night. So he puts is feet up and lays back.
Anterior circulation on interior (leg) and middle circulation on outside (arm/face).
* MCA infarct also comes with aphasia. Broca aphasia is expressive aphasia with intact understanding. Broca is
broken speech. Wernicke aphasia is wordy, nonsensical speech, unable to understand. Aphasia occurs with dominant
hemisphere stroke. Most people are left-hemisphere dominant.
* Basilar artery provides blood supply to cerebellum, pons, and brainstem. Brainstem (medulla/pons/midbrain) is
where cranial nerves are. So, patient could have sudden onset of diplopia, blurry vision, dysphagia, focal cranial
nerve palsies. Cerebellar signs include ataxia and vertigo.
* With posterior circulation (basilar artery), deficit is contralateral if above decussation and ipsilateral if below. A
cross syndrome is a cranial nerve deficit on one side, and a motor deficit (hemiparesis, ataxia) on the opposite side.
Cross syndrome is most likely posterior circulation stroke.
* A cranial nerve III deficit (cannot adduct eye) on one side and hemiparesis on the other is Weber syndrome.
* A cranial nerve III deficit on one side and ataxia on the other is Benedikt syndrome.
* A sensory loss on one side of the face with contralateral sensory loss on the body is Wallenberg syndrome.
* PCA supplies occipital lobe, giving us the ability to see. PCA ischemia comes with hallucinations, visual loss.
* 56yo woman is brought to the ED by her daughter, complaining of sudden onset of right upper extremity weakness
that started while she was watching television in the morning. Her daughter became concerned when her mother was
unable to talk in response to her questions. On neurologic exam, you note right upper extremity weakness with
pronator drift and right facial palsy. When you question the patient, she seems to understand what is being said but
she cannot clearly respond.
* Patient has arm/face hemiparesis and Broca aphasia. Most likely diagnosis is MCA infarct (left side). Next step in
management is head CT scan, not aspirin, not clopidogrel, not ticlopidine, not heparin, not transthoracic echo, not
transesophageal echo, not tPA. Now, do we give contrast or no contrast? We are trying to distinguish between a
bleed and ischemia, because these two have different management. Answer is head CT without contrast. We are
looking for the absence of blood. If there is no bleeding on the non-contrast CT, we assume ischemic stroke.
* Most sensitive (accurate) test for diagnosing ischemic stroke is MRI. MRI most sensitive for posterior fossa
lesions. CT scan without IV contrast is most sensitive test for hemorrhagic stroke.
* With ischemic stroke, look for reversible risk factors such as cardiac thrombosis (echocardiogram), carotid artery
stenosis (carotid artery duplex, mainly for anterior circulation stroke), atrial fibrillation (24h Holter monitor).
* Hemorrhagic stroke is managed by supportive care and consulting neurosurgery (poor prognosis). For ischemic
stroke, we can think about giving aspirin, clopidogrel, heparin, tPA, ticlopidine.
* If patient has acute onset focal neurologic deficit, CT shows no bleeding, you are sure it is ischemic stroke, and
symptom onset is within 3 hours, then answer is tPA/PLAT (tissue plasminogen activator).
* If patient was sleeping and woke up with symptoms, tPA should not be given even if you get the CT minutes later.

DO NOT DISTRIBUTE -8-


Study Notes – Internal Medicine James Lamberg 01Feb2010

* Must document time of onset and CT scan with diagnosis before giving tPA. If you give tPA past the 3-hour
window, you’re only increasing their risk of bleeding. So the risk to benefit ratio is not favorable after 3 hours.
* Contraindications for tPA therapy are bleeding (e.g. GI bleed, urinary tract bleed) within past 21 days, surgery
within past 14 days, BP > 185/110, platelet count < 100,000, PT > 15sec, ischemic stroke or head trauma within 3
months, intracranial bleed ever in the past.
* Aspirin is used in ischemic stroke for secondary prevention. The benefit of aspirin in acute stroke is not as good as
with acute myocardial infarction, but we give it anyway.
* Clopidogrel is used if the patient has failed aspirin therapy, meaning the patient is on aspirin after an ischemic
stroke then has another ischemic stroke. Clopidogrel side effects include TTP and neutropenia.
* Ticlopidine is never the answer due to side effect, which are TTP and neutropenia. The difference between
clopidogrel and ticlopidine is that ticlopidine causes the side effects (TTP, neutropenia) more often.
* If the patient is allergic to aspirin, failed aspirin therapy, or is allergic to clopidogrel, add dipyridamole.
* Heparin reduces the rate of recurrent CVA. For every stroke you prevent, you cause one complication (e.g. bleed).
So the only time you give heparin is when you have a higher risk of CVA. There is a higher risk of recurrent CVA if
the patient has ischemic stroke with atrial fibrillation, basilar artery thrombosis, or stroke in evolution.
* Stoke in evolution is when symptoms are getting worse, a loss of brain function occurs with brain cell death.
* Say you do a workup on the 56yo lady that has right-sided facial/arm hemiparesis. Carotid duplex finds 80%
stenosis of the right internal carotid artery. Answer is not surgery. Indication for carotid endarterectomy (CEA) is
>70% stenosis with symptoms. Symptoms mean you have to be able to blame a TIA or a stroke on the stenosis. If
the patient’s symptoms were due to the carotid artery stenosis, the stenosis would be on the left side (not the right).
* When looking at a CT scan to evaluate a suspected CVA, you’re looking for the presence of white material in the
parenchyma of the brain. In ischemic stroke, the tissue will look darker than the surrounding tissue.
Seizures
* 29yo man is brought to the ED by ambulance after his mother found him convulsing in his bedroom. The patient’s
mother said that her son was unable to respond to her frantic cries during the convulsion and describes jerking
motions that become more frequent then stopped after about 1 minute. The mother says he was tired and lethargic
for 20 minutes after the episode. She then called the ambulance to bring her son to the hospital.
* What is the most likely diagnosis? Answer is seizure. Differentiate seizure from syncope, say due to arrhythmia.
This patient was having convulsions (tonic-clonic movements). Patients who syncopize can have tonic/clonic
movements. Urinary or bower incontinence is an important part of the history, but not necessarily specific. Bite
marks on the tongue may imply seizure, but not specific either. The most specific thing on history for seizure is the
post-ictal state. Patients who have syncope and come out will not be lethargic, tired, or with achy muscles. Syncope
has rapid recovery within minutes or seconds after unconsciousness. Here, the patient was tired/achy for 20minutes.
So this is the clear differentiation.
* Urinary and bowel incontinence plus bite marks are seen more commonly in seizure than syncope. However, the
post-ictal state still is the most specific symptom.
* A seizure is defined as random firing of neurons in the brain. A seizure is considered a complaint, like a patient
coming in with chest pain; there is a large differential diagnosis for seizure.
* Ask yourself if there is an underlying cause for the seizure. Ask if the patient has a history of epilepsy. Do not
assume that any seizing patient has a diagnosis of epilepsy.
* Seizure differential mnemonic: VITAMINS.
* Seizure: vascular (stroke, bleed, AVM), infection (encephalitis, meningitis), trauma, autoimmune (vasculitis,
SLE), metabolic (electrolytes, glucose, drugs), idiopathic, neoplastic (metastatic cancer, primary tumors), s for Psi or
psychiatric (patient faking).
* If you’re thinking vascular disease, look for risk factors and sudden onset of neurologic focality. Suspect infection
if the patient has seizure with fever, or nuchal rigidity, or photophobia. Suspect autoimmune if history involves a
rash, purpura, low-grade fever with weight loss, arthritis, SLE stigmata, positive ANA. For metabolic, look at things
like low sodium, high sodium, low calcium, low oxygen, low glucose, low magnesium. If a young woman with
breast cancer has a seizure, think about metastatic to the brain.
* The next step in management is ABCs: airway, breathing, and circulation. Then benzodiazepine.
* 29yo man comes in with new-onset seizure witnessed by his mother, convulsing with a long post-ictal state, and
the patient is already intubated. Blood pressure and circulation are intact. Patient is continuing to seize. What is the
best initial treatment now? Answer is give lorazepam or diazepam, for acutely seizing patient.
* Always look for a secondary cause. If you identify a secondary cause, treat it, hoping seizures resolve. So if
patient comes in seizing with sodium of 106, treatment would be focused on hypertonic saline. If patient came in
hypoxemic and seizing, treatment would be focused on intubation and high-flow oxygen (after ABCs).

DO NOT DISTRIBUTE -9-


Study Notes – Internal Medicine James Lamberg 01Feb2010

* If patient is continuing to seize without regaining consciousness between, this is status epilepticus. Management is
lorazepam or diazepam.
* Say you give the benzo and the patient is still seizing. Then what do you give? Answer is phenytoin or
fosphenytoin. Note, giving phenytoin IV is not recommended due to lack of solubility and resultant precipitation,
give fosphenytoin instead.
* What if patient continues to seize after giving phenytoin or fosphenytoin? Give phenobarbital.
* Next step after phenobarbital? Answer is midazolam and propofol (anesthesia).
* Seizure Meds: ABC, lorazepam/diazepam, phenytoin/fosphenytoin, phenobarbital, and last midazolam/propofol.
* Seizures are categorized into partial, generalized, and complex vs. simple. Categories help determine what
medication to give. A complex partial seizure is treated differently from a complex generalized seizure.
* A partial seizure only affects one part of the brain. So patient might have shaking/jerking of the hand, or the foot,
or the leg. A generalized seizure affects the entire cortex. Either the seizure starts as generalized, or a partial seizure
turns into a generalized seizure. A complex seizure implies loss of consciousness. A simple seizure has no loss of
consciousness. An atonic seizure means lack of tone, so patient has “drop attacks.” An absence seizure is the
opposite of an atonic seizure, so patient keeps postural tone but brain shuts down so the patient is not paying
attention (no consciousness) and likely just blinking. Myoclonic seizures involve muscle jerks.
* What is the best test to identify abnormal neural activity that predisposes to a seizure? Answer is
electroencephalogram (EEG). So patient has first time seizure with no exact etiology, order an EEG.
* What if EEG is negative, when do you start treating an idiopathic seizure? Answer is with recurrent seizures.
* So patient has normal neurologic exam, negative EEG, negative family history, and one seizure. We follow the
patient. Once the patient has another seizure, we start treatment. If the patient has positive family history, we would
start right away at the first seizure.
* Medication for partial seizure (even one that becomes generalized) is carbamazepine or phenytoin. If
carbamazepine or phenytoin are not answers, pick valproic acid.
* Medication for generalized seizure is valproic acid or lamotrigine.
* Medication for absence seizure is ethosuximide.
* Medication for atonic or myoclonic seizure is valproic acid.
* Medication for unidentifiable seizure is valproic acid, as it is the most widely effective seizure medication.
Parkinson Disease
* A 56yo man is brought in by his wife for evaluation of a resting tremor that she noticed recently. She also states
that her husband has been moving very slowly as of late. When questioned, the patient states that he feels fine and
does not know why his wife is dragging him from doctor to doctor. His past medical history is significant for mild
hypertension treated with a thiazide diuretic. Physical exam finds a resting tremor noted in his right hand, and when
walking the patient is stooped forward making small steps. You note cogwheel rigidity in his right upper extremity
with a positive Myerson sign (patient unable to resist blinking with glabellar tapping).
* Differential diagnosis for tremor includes Parkinson disease, essential tremor, and cerebellar disease. In Parkinson
disease, the tremor occurs at rest and resolves with movement. In cerebellar disease, the tremor occurs primarily
with movement (intention tremor). Exam for cerebellar disease would include finger-to-nose and heel-to-shin tests,
with the patient have a compromised ability to reach the object. In Parkinson disease, the patient will have pill-
rolling tremor at rest. In essential tremor, the tremor occurs and worsens with movement, there is a family history
usually, and there are no other stigmata of cerebellar or Parkinson disease.
* So in the 56yo man with resting tremor, slow movement (bradykinesia, paucity of movement), history of falls
(postural instability), and cogwheel rigidity (arm feels like cogs on a wheel, moving in distinct steps), what is the
most likely diagnosis? Answer is Parkinson disease. So, diagnosis of Parkinsonism is clinical.
* Normally when you walk and turn it is a smooth movement. With postural instability, patient turns in wide circle.
* Causes of Parkinsonism are drugs (antipsychotics, metoclopramide, MPTP: an unintentional byproduct of the
recreational drug MPPP), tumor, bleed or stroke, CO poisoning, cobalt, manganese. So ask yourself about secondary
causes. Is this patient diabetic and taking metoclopramide for autonomic neuropathy with gastric paresis?
* Most cases of Parkinson disease are idiopathic, meaning we cannot identify a cause.
* Parkinsonism is defined as death of dopaminergic cells in the substantia nigra.
* The drugs that cause Parkinson are antipsychotics, which are dopamine antagonists. You can decrease the amount
of dopamine by antagonizing it (decreasing it directly) or by increasing acetylcholine. Acetylcholine acts to inhibit
dopaminergic tone in the brain. So treatment is to either give dopamine to provide what is lost, or to take away
acetylcholine so the dopamine in the brain can work more effectively.
* Dopamine agonists are bromocriptine, carbidopa/levodopa, pergolide, pramipexole, and ropinirole. Carbidopa
inhibits the conversion of levodopa to dopamine in the periphery, so that the levodopa can reach the brain and be

DO NOT DISTRIBUTE - 10 -
Study Notes – Internal Medicine James Lamberg 01Feb2010

converted to dopamine there; thus we can give less levodopa and get the same effect.
* Selegiline (MAO-B inhibitor), COMT inhibitors, and amantadine increase dopamine amount/effect also.
* Acetylcholine blocking with trihexyphenidyl primarily.
* So how do you choose what to answer as the best initial treatment for Parkinson disease? First question to ask your
self is what is the functional status of the patient? Meaning, how do they function on a day-by-day basis?
* Functional patients get amantadine or anticholinergic (e.g. trihexyphenidyl).
* Non-functional patients get carbidopa/levodopa, the most effective treatment for Parkinson disease. We don’t give
carbidopa/levodopa to functional patients because it has the most serious side effects.
* Always ask how much function you are getting from the treatment at the expense of the side effects.
* Side effects are psychosis, hypotension, and acute GI upset. After long-term therapy on carbidopa/levodopa,
patients get response fluctuations. Response fluctuations are the “on/off phenomenon,” akinesia (restlessness), and
dyskinesia (abnormal movements). On/off phenomenon is quite distressing to patients.
* Say patient is functional and just has a tremor, pick amantadine if patient is > 65yo. Pick trihexyphenidyl if patient
is < 60yo. We avoid anticholinergic medications in elderly patients because they can become confused, have urinary
retention, dry mouth, dry eyes, more so than younger patients.
* Answer dopamine agonists (e.g. pramipexole, ropinirole) to treat response fluctuations for patients that are taking
carbidopa/levodopa. You can also give a COMT inhibitor or selegiline.
* If patient is on carbidopa/levodopa and it isn’t enough, add the dopamine agonist.
* The most preferred dopamine agonists are the newer ones, like ropinirole (not selegiline or COMT inhibitor).
* Say patient has stigmata of Parkinson and you are asked what medication is thought to arrest the progression of
Parkinson disease, the answer is selegiline.
Huntington Disease
* 34yo man comes to your clinic for evaluation of strange spontaneous movements that have been occurring lately.
Recently while sitting at a family dinner, the patient experienced uncontrolled grimacing with grunting. His family
history is significant for his father who died at age 41 of dementia.
* Most likely diagnosis is Huntington disease. Know that patients will present with abnormal movement (chorea),
abnormal behavior, loss of inhibition, changes in personality, and a family history of a similar thing happening to a
first-degree relative (Huntington is autosomal dominant).
* Clinical diagnosis is presence of chorea with personality changes, usually in a patient 30-40yo with a positive
family history. True diagnosis done with genetic analysis, looking for chromosome 4p CAG repeat.
* Treatment is supportive care; there is no formal treatment.
Multiple Sclerosis (MS)
* A 32yo woman comes to the ED with numbness and tingling in her right hand. She states that her symptoms began
several days before admission and have progressively worsened over the last several hours. When asked, she states
that three years ago she had an episode of seeing double that lasted two days that resolved on its own. Physical exam
is significant for hyperreactive reflexes bilaterally in the lower extremities. You also note increased spasticity in her
lower extremities.
* Most likely diagnosis is multiple sclerosis (MS).
* Differential includes MS, CVA, brain tumor, cervical spinal disease, and carpal tunnel syndrome.
* CVA less likely because she is 32yo with no other listed risk factors.
* Brain tumor less likely as there is no specific tumor location giving lower extremity weakness along with diplopia.
* Cervical spinal disease or carpal tunnel less likely because they do not explain the diplopia.
* The essential point for clinically suspecting MS is a patient with multiple neurologic deficits that are separated by
space (anatomically) and by time (temporally).
* Testing includes MRI with gadolinium, CSF for oligoclonal banding, and olfactory/visual evoked potentials
looking for abnormal transmission.
* What is the most sensitive/accurate test? Answer is MRI or brain and spine, 85-95% sensitive. Looking for
multiple lesions and paraventricular lesions consistent with MS.
* The best initial test is also MRI, not CT scan, not lumbar puncture.
* If MRI is inconclusive or equivocal for the clinically suspected MS diagnosis, then get the lumbar puncture.
* Olfactory/visual evoked potentials are not used much today with the development of MRI.
* Relapsing/remitting disease is a form of MS where patient goes through waxing/waning episodes of symptoms.
* Relapsing/remitting disease can become progressive (secondary progression). Primary progressive disease has
worsening MS symptoms right from presentation, least common, and worse prognosis.
* Treatment for acute exacerbation of MS is IV high-dose steroids with a 4-week taper on oral prednisone. Steroids
do not slow down the progression of the disease, but they help symptomatically for the disease relapse.

DO NOT DISTRIBUTE - 11 -
Study Notes – Internal Medicine James Lamberg 01Feb2010

* Medications useful to arrest the progression of MS are interferon beta 1a, interferon beta 1b, or glatiramer acetate.
None of these three is better than the other. They have been shown to help with relapsing/remitting forms, but not in
primary progression. No medication has been shown to help with primary progressive disease.
* Treatment for spasticity in MS is baclofen.
* Treatment for fatigue in MS is amantadine.
* Treatment for urinary incontinence in MS is oxybutynin.
* Treatment for urinary retention in MS is bethanecol.
Dementia
* A 67yo woman is brought to your clinic complaining of forgetfulness. She states that recently she has been
forgetting telephone numbers and cannot remember the name of her mailman who she has known for 25 years. Her
past medical history is significant for hypertension, coronary artery disease, and high cholesterol. Physical exam is
unremarkable.
* With memory loss, consider dementia and ask yourself if there is a reversible cause.
* Reversible causes: hypothyroidism (check TSH), vitamin B12 deficiency (check B12 level), frontal lobe neoplasm
and chronic subdural hematoma (check for focality on exam or trauma history), syphilis (check RPR: rapid plasma
reagin), uremia and cirrhosis (check creatinine, LFTs, physical exam), central or obstructive sleep apnea (look for
obesity, speak with spouse), HIV.
* Non-reversible causes: Creutzfeldt-Jakob disease and prion disease (check for myoclonus, rapid dementia course
usually weeks to months), Lewy body disease (delirium-like course, waxing/waning), multi-infarct dementia
(stepwise progression, tie temporally dementia onset with CVA, difficult to differentiate from Alzheimer).
* Most common cause of dementia is Alzheimer disease. In Alzheimer disease, there is initial memory loss that
becomes chronic and gradual with relative preservation of social function and personality until late in the disease.
* Pick disease (frontal lobe degeneration) has personality changes initially, such as agitation, violence.
* Dementia is generally memory loss plus some other deficit in cognitive function, such as concentration, praxis, or
executive function. This is where the mini-mental status exam (MMSE) is important.
* Patient presents with memory loss. Perform MMSE to find dementia. Then rule out or treat reversible causes.
Then rule out non-reversible causes like Pick (personality changes first), CJD (myoclonus, rapid progression), multi-
infarct dementia (temporal tie to CVA), and Lewy body (delirium-like). Now, we diagnose Alzheimer disease.
* Treatment for Alzheimer disease was tacrine, but no longer used as the drug of choice due to liver toxicity.
* Medication of choice for Alzheimer now is donepezil. Do not pick tacrine for first line.
Vertigo
* A 53yo woman is brought to the ED complaining of dizziness. The patient describes walking to her bathroom and
experiencing a sudden feeling of nausea. She managed to reach the bathroom where she vomited once and fell to the
floor a second time. She was unable to get up off the floor and called 911. She describes the feeling of the room
spinning around her even though she realizes she is not moving.
* Most important question is to identify what the patient means by dizziness. “Dizzy” is very nonspecific and does
not have much significance medically. Does the patient mean they are having vertigo or do they mean pre-syncope?
* Pre-syncope associated symptoms are feeling like going to black out, light headed, palpitations, chest pain,
shortness of breath. This implies cardiac disease.
* Vertigo is the sensation of movement in the absence of movement. Patient may say the room is spinning around
me, the earth is rolling in front of my feet, like they are falling forward.
* Differentiating between vertigo and pre-syncope is a clinical determination.
* Next question after determining vertigo is if it is central (in the CNS, e.g. MS, posterior fossa tumor, medication)
or peripheral (ear semicircular cannals). For central, we do MRI, for peripheral, we do symptomatic treatment.
* Central vertigo is usually chronic, neighborhood signs (cranial nerve deficits via brainstem), pure nystagmus (in
one direction, usually vertical), nystagmus non-suppressible with fixation, multi-directional nystagmus
* Peripheral vertigo is usually acute, hearing loss, tinnitus, mixed nystagmus (usually horizontal but with another
component), nystagmus suppresses with fixation, uni-directional nystagmus
* Posterior fossa imaged more effectively with MRI than with CT scan.
* Ménière disease is a triad of (peripheral) vertigo, hearing loss, and tinnitus unrelated to head movement. Treatment
of choice is diuretics, low salt diet, and surgical decompression if medical management fails. Disease is thought to
be caused by swelling in the semicircular canals.
* Benign positional paroxysmal vertigo (BPPV) is peripheral vertigo of sudden onset related to movement of the
head. Patient may say I move my head then 5-10 seconds later I get dizzy. Physical exam can reproduce symptoms
via Dix-Hallpike test (head movement testing). Etiology is thought to be an otolith in the semicircular canals.
Treatment of choice is movement exercises to try and move the otolith out of the canal.

DO NOT DISTRIBUTE - 12 -
Study Notes – Internal Medicine James Lamberg 01Feb2010

* Labyrinthitis is peripheral vertigo that occurs when vertigo follows an upper respiratory tract infection. Treatment
is meclizine (antihistamine) or diazepam in severe cases.
* So, if you find central vertigo signs on physical exam, do MRI imaging of the posterior fossa. If you identify
peripheral vertigo, seek Meniere, BPPV, or labyrinthitis with their treatments listed.
* If you cannot identify a disease, give meclizine initially then diazepam secondary.
Headache
* A 32yo woman comes into the office complaining of a headache that started two days ago. She locates her
headache to the right side of her head and describes it as pulsating and throbbing in quality. The headache is
worsened by walking up stairs or around the block. She admits to nausea but denies vomiting. She also states that
loud noise and bright light exacerbate her pain. On exam, there is no focality on physical exam, she has had these
headaches a few times before. Most likely diagnosis is a migraine.
* Ask yourself, what is the likelihood that this headache is secondary to some serious underlying pathology? Is this a
brain tumor? Meningitis? Subarachnoid bleed? Do we need to diagnose and intervene immediately?
* History/physical implying serious underlying cause: first time headache with severe pain, fever, progressively
worsening headache, symptoms aggravated by cough, worse in the early morning, aggravated by valsalva maneuver,
nuchal rigidity, “worst headache of my life” (subarachnoid), thunderclap headache (time to peak pain was seconds),
focality on CNS exam, headache occurs after vomiting, any headache starting at age > 55yo.
* Primary headache disorders include migraine, cluster, and tension.
* Migraines are associated with triggers, such as eating a specific food (e.g. chocolate), sleeping too little, stress, or
during menstruation. A trigger is very specific for migraine.
* Pulsatile headaches occur in cluster and migraine headaches (both vascular).
* Photophobia and phonophobia (ligyrophobia) classically occurs in migraine but may occur in cluster also.
* Cluster headaches are usually unilateral (like migraine), no trigger, has rhinorrhea, red eye, possibly Horner
syndrome (ptosis, miosis, anhidrosis).
* Tension headaches are triggered by stress but not foods or emotion, are bilateral, vice-like, radiating to neck or
back of the head.
* Migraines peak within 4-72 hours from onset of pain (gradual onset). Cluster headaches peak usually within 5
minutes (quick onset) and only last 45-90 minutes.
* Clusters happen 2-3 times per day over a 4-8 week period; occurring in clusters.
* Management for tension headache is analgesics, like acetaminophen or NSAIDs.
* Management for cluster headaches starts with 100% oxygen. Next line is sumatriptan. Prophylaxis for cluster
headaches includes lithium, prednisone, or verapamil.
* Management for mild migraine headache (no nausea or vomiting), give NSAIDs. If migraine is moderate to
severe, abortive therapy is sumatriptan or ergot alkaloid. Prophylaxis for migraines is indicated when patient has
more than 3 headaches per month. Prophylaxis is propranolol, timolol, or methysergide (generally not used because
it causes retroperitoneal fibrosis).
* Never give sumatriptan to a patient with a history of coronary artery disease. Migraines are generally caused by
vasodilation and sumatriptan constricts the vessels. So if a patient has a 70% left main coronary lesion and you give
them sumatriptan, you just gave them an MI. Do not give it even if you have high suspicion of CAD.
Acute Inflammatory Demyelinating Polyradiculoneuropathy (AIDP)
* A 46yo man is brought to your office complaining of rubbery legs. The patient states that his symptoms began two
days ago. Approximately three weeks ago the patient states that he experienced several episodes of diarrhea that
resolved spontaneously. On neurological exam, the patient is noted to have bilaterally lower extremity weakness
with loss of reflexes.
* Most likely diagnosis is Guillain-Barré syndrome, an AIDP. CIDP (chronic) has a slower course, seen in HIV.
* The distribution of weakness is important when differentiating between Guillain-Barré, myasthenia gravis (MG),
and botulism. In Guillain-Barré, distribution starts in distal areas and moves proximal. Paresthesias (tingling) and
autonomic instability (labile blood pressure, sweating) are also seen.
* Campylobacter jejuni infection (gastroenteritis) associated with Guillain-Barré syndrome (GBS).
* Hyporeflexia seen in any peripheral neuropathy.
* The best initial test for the diagnosis of GBS is lumbar puncture, looking at CSF for high protein that is not
accompanied by pleocytosis (a high number of cells). So you’re looking for high protein and few cells.
* The best test (most accurate) for GBS is an electromyogram (EMG).
* Best initial treatment for GBS is intravenous immunoglobulin. You can also do plasmapheresis. IVIg and
plasmapheresis are equivalent in their effect. Do not give prednisone or systemic steroids in the treatment of acute
Guillain-Barré syndrome. IVIg is easier to give than arranging for plasmapheresis, so IVIg best initial treatment.

DO NOT DISTRIBUTE - 13 -
Study Notes – Internal Medicine James Lamberg 01Feb2010

Myasthenia Gravis (MG)


* A 35yo woman comes to the clinic complaining of double vision that seems worse near the end of the day. The
patient also complains of difficulty chewing meat and other hard foods. She notices that her symptoms improve
following a good night sleep. On neurologic exam you notice snarling appearance when the patient is asked to smile
and a nasal tone to her speech. You also note weakness of the upper extremities when the patient is asked to clench
her fists around your fingers repeatedly.
* Most likely diagnosis is myasthenia gravis (MG).
* In MG, there typically is not a distal to proximal movement. Patients complain of easy fatigability, especially in
the muscles of the throat and eyes (dysphagia, ptosis, diplopia). Symptoms are not acute either.
* MG is defined as muscle fatigue after repetitive motion of chronic onset with preferential involvement of the
ocular and the pharyngeal muscles.
* The best initial test for diagnosing myasthenia is serology for the anti-acetylcholine receptor antibody. A tensilon
test is not the initial test. In the context of clinical suspicion of myasthenia, anti-acetylcholine receptor antibody is
extremely specific for the diagnosis.
* A positive tensilon test is not specific for the diagnosis; false positives seen in other diseases like amyotrophic
lateral sclerosis (ALS, Lou Gehrig disease).
* Only answer tensilon testing when there is no answer choice for acetylcholine receptor antibody.
* The best (most accurate, most sensitive) test for the diagnosis is electromyography (EEG) looking for a decrease in
the action potential spike on repetitive stimulation.
* MG is essentially an autoimmune disease against the acetylcholine receptor.
* Treatment is symptomatic for weakness and for the disease with autoimmune suppression.
* Symptomatic treatment is with anticholinesterase medication (neostigmine, pyridostigmine) to raise the level of
acetylcholine in the synapse, improving the weakness.
* To treat the autoimmune disease, give prednisone, azathioprine, IV immunoglobulin, plasmapheresis, and
thymectomy. Prednisone is best initial immunosuppressive because it takes 1-3 months to have an effect. Patients
may initially have a worsening of their disease. Azathioprine takes 3-6 months to have an effect.
* If acute myasthenic crisis, use IVIg or plasmapheresis to improve the patient’s crisis immediately and acutely.
* In patients with GBS or MG, you must follow their respiratory status. The diaphragm is a skeletal muscle and
when the disease involves the diaphragm the patient can go into respiratory failure. So, always consider the
possibility of an intubation and respiratory support to get them through the crisis.

Kaplan Videos – Pulmonology with Dr. Asher Kornbluth, MD


Read the last line of the question stem to help determine what organ system the question is asking about. This will
help you develop a better differential diagnosis as you read the question.
Pulmonary Function Tests
* A pulmonary function tests (PFT) is generally used to differentiate between obstructive and restrictive disease.
There may be a great deal of overlap between these.
* Tidal volume is normal in-and-out respiration at rest. Good rule of thumb is 10mL/kg, so 700mL for a 70kg
patient. Vital capacity is deep breath then exhaling maximally, does not empty lungs completely. Residual volume is
left over lung volume after vital capacity. Vital capacity plus residual volume is total lung capacity. Residual volume
is a difficult measurement to make and not done in clinical practice, so you never know the exact total lung capacity.
* In restrictive pattern, all lung volumes are decreased. Therefore total lung capacity is decreased. Lungs are
restricted and cannot expand due to disease such as sarcoidosis, interstitial fibrosis, and chronic tuberculosis.
* In obstructive pattern, there is an obstruction in getting air out of the lungs; a hallmark of asthma and chronic
obstructive pulmonary disease (COPD). Residual volume will be higher because you cannot expel all of it. Over
years, the total lung capacity will increase with increased residual volume. This is what causes the large AP
diameter, giving the barrel chest appearance. Measures of air outflow are reduced in obstructive disease.
* FEV1 measured by having patient take deep breath in then breath out as fast as possible, with the forced expiratory
volume measured over 1 second.
* FVC is the forced vital capacity, basically the same as vital capacity.
* FEV1/FVC ratio is decreased in obstructive disease, mostly because FEV1 is low. So FEV1 and FEV1/FVC low.
* FEV1/FVC ratio is normal in restrictive disease, with low FEV1 and low FVC.
* FV25-75 is forced volume between 25% and 75% during a vital capacity. FV25-75 reduced in obstructive.
* Residual volume high in obstructive lung disease, low in restrictive lung disease, but RV not easy to measure.
Alveolar-Capillary Membrane Diffusion
* Diffusion capacity is a measure of alveolar capillary membrane, not an easy measurement to do. Alveolar-

DO NOT DISTRIBUTE - 14 -
Study Notes – Internal Medicine James Lamberg 01Feb2010

capillary membrane gets destroyed/fibrosed in emphysema. If the DLCO (diffusion of the lung carbon monoxide) is
low, implies destruction of the alveolar-capillary membrane, so emphysema or ARDS (acute respiratory distress
syndrome). ARDS patients are generally in the ICU on a respiratory so a DLCO isn’t measure.
* In bronchitis DLCO may be normal, in emphysema DLCO may be low.
* Simpler way to measure gas exchange effectiveness is by measuring alveolar-arterial difference in oxygen, the
AaDO2 or Aa gradient. This involves getting an arterial blood gas (ABG) after performing an Allen test to ensure
ulnar collateral circulation (not sensitive but still done). On ABG, lab gives you pH, PCO2, PO2.
* Say PO2 80mmHg (normally 90-100). Alveolar O2 at room air (21% O2) is 150 - 1.2*PCO2.
* Normal PCO2 is 40, so Alveolar O2 would be 150 - 1.2*(40) = 102. Difference is achieved by subtracting what
O2 you got in the blood gas (80mmHg), so 22 here. This is mildly elevated as normal is 5-15.
* If there is a pulmonary process affecting the membrane, A-a gradient may be as high as 40.
* Say our ABG comes back as 7.4/20/100. Alveolar is 150-1.2*20 = 126. 126-100 (PO2) gives 26 for Aa gradient.
* Think about this physiologic process. The patient is breathing very fast to get PCO2 down to 20, so you know
there is some problem. That is why measurement of PO2 does not tell you what kind of gas exchange you have.
* So in summary, A-a gradient tells you what is going on for gas exchange at the alveolar-capillary membrane.
* Given 7.4/36/96. Find Aa gradient. 150-1.2*36=107. 107-96 is 11 for Aa gradient (normal range).
Solitary Pulmonary Nodule
* A solitary pulmonary nodule is a single nodule less than 6cm. Lung cancer is rarely cured. The only time you can
cure is if you have a solitary cancer and resect before it metastasizes.
* So you found a < 6cm nodule on chest x-ray. What are the criteria for just following this nodule? Young patient
(<50yo) and non-smoker. If this is the case, repeat the chest x-ray every 6 months for 2 years. If the nodule has not
changed size, we know it is benign. Tell patient to take CXR with them next time they go for an x-ray.
* If you are looking for calcification within a nodule, do a chest CT scan due to increased sensitivity.
* Popcorn appearance (scattered areas) of calcification within nodule, think hamartoma (e.g. hair, tooth).
* Central or bull’s-eye calcification within nodule, think granulomatous disease (e.g. tuberculosis).
* Eccentric (near edge) single area of calcification within nodule, think possible malignancy.
* Say patient is 60yo and 50pack*year smoker. You find a 2cm nodule on CXR. Next step is do a biopsy, most of
the time by open lung biopsy. Usually nodule is not central enough to reach with simple bronchoscopy. If the biopsy
shows malignant cells, next step is surgery to remove. If benign, follow with CXRs.
Pleural Effusion
* The pleura covering the organ (the viscera) is the visceral pleura. The pleura outside that is the parietal pleura,
with fluid between the two pleurae. When fluid increases between these two, this is a sign of a pleural effusion.
* Early sign of pleural effusion is loss of costophrenic angle at the diaphragm, requires about 300mL of fluid.
* Lay patient on the side of the effusion and do a lateral decubitus x-ray. If the effusion is free-flowing, the fluid will
layer out into the dependent portion. This is useful to know to determine if you will get fluid via thoracentesis. If the
fluid were trapped, we have to hit the trapped portion itself.
* Pleural effusions classified into transudate and exudate. In general, exudate is seen in sicker patients. Transudate is
seen in high-pressure differences (e.g. CHF) or low blood oncotic pressure (e.g. nephrotic syndrome, cirrhosis).
Cirrhosis causes hypoalbuminemia (protein not created) typically resulting in right pleural effusion.
* To differentiate, need pleural LDH and protein, with comparison to serum LDH and protein. Transudate has low
LDH (<200) and low protein. Pleural to serum LDH < 60% and pleural to serum protein < 50%.
* Transudate typically caused by systemic diseases. Exudates typically caused by lung diseases.
* Exudate: pleura has some process that causes it to exude fluid. The larger the effusion, the safer the thoracentesis.
* Say we do the tap and the LDH is 300 and protein is 3grams (high). This is an exudate.
* Exudate causes include cancer, tuberculosis, pulmonary embolus with infarction, pneumonia. If infected material
exudes out (parapneumonic), we have a complicated effusion and need a chest tube to drain else it scars and the
patient loses lung volume. This scaring can happen quickly.
* Complicated exudative effusion (requiring chest tube) includes infection with pH < 7, pus on tap, polys on gram
stain, and low glucose in fluid. Next step is chest tube, not antibiotics, not intubation, not further diagnostic studies.
* Parapneumonic effusion leaks fluid but no low pH, no pus, no polys on gram stain, not low glucose. So this is an
uncomplicated parapneumonic effusion. This patient does not need a chest tube. So patient has fever, pneumonia,
and effusion. Next step is put a needle into the space to determine if it is complicated. If not, just treat pneumonia.
* Empyema is an effusion with pus in the space, requires a chest tube.
Atelectasis
* A 62yo man is dyspneic 24hours after cholecystectomy. His respiratory rate is 22, and pulse is 112, has a mild
fever, and decreased breath sounds are noted in the left lower lobe. CBC shows leukocytosis of 27,000.

DO NOT DISTRIBUTE - 15 -
Study Notes – Internal Medicine James Lamberg 01Feb2010

* Patient is post-op with anesthesia medication causing decreased lung expansion, is on narcotic meds and likely not
taking in deep breaths because of pain. Most likely diagnosis is atelectasis.
* Atelectasis is collapsed alveolar airways, lung itself is not condensed on itself.
* Massive ascites pressing up on lungs and impairing ability to take deep breath can cause atelectasis.
* Broken ribs causing pain on inspiration can cause atelectasis.
* This can give rhonci or crackling at the bases, perhaps a little wheezing.
* You can get mild fever with atelectasis. Post-op surgical “W”s of fever: Wind (atelectasis), Water (UTI), Walking
(DVT), Wound (incision infection), Wonder drugs (drug causes) for days 1, 3, 5, 7, and 9.
* Next step in diagnosis is chest x-ray. Treatment is inspired spirometry breathing device.
Chronic Obstructive Pulmonary Disease (COPD)
* Patient has COPD, is a non-smoker, liver problems, and has relatives that died of lung or liver problems. Think
alpha-1 antitrypsin deficiency. Patient can have either liver or lung involved, not necessarily both.
* Treatment is symptomatically, recurrent antibiotics, hoping they do not need a lung or liver transplant.
* COPD patients can develop pulmonary hypertension, giving a loud second sound on heart exam (P2). During
inspiration, right-sided heart pressures increase. So here the second heart sound will get louder on inspiration.
Patient may have a right-ventricular heave, and ECG may show right-ventricular hypertrophy.
* Worry that this patient will develop cor pulmonale. This is similar to CHF in a patient developing left ventricular
failure due to hypertension.
* Patient with COPD has obstruction so you hear increased wheezing, large AP diameter (barrel chest), and history
including smoking, past episodes of bronchitis.
* Patient with chronic bronchitis will have increased markings on chest x-ray. Patient with emphysema will have
blebs that can become bullae, flattening of the diaphragm, widened silhouette so heart looks smaller, breath and
heart sounds may sound very distant.
* Pulmonary function tests will be diminished FEV1 and FEV1/FVC ratio.
* DLCO test can help differentiate between bronchitic patient and emphysematous patient. There is destruction of
the alveolar-capillary membrane in emphysema (low DLCO) but not bronchitis.
* In an asthmatic, we give them beta2 agonists acutely and try to keep them well between attacks with steroids.
* Ipratropium tends to work better in COPD than asthma for bronchodilitation.
* Beta2 agonists help in COPD. Antibiotics for frequent bronchitic infections; They get pneumococcus, h. influenza,
moraxella catarrhalis. We use sulfa antibiotics, penicillin antibiotics, cephalosporins, sometimes tetracycline. We try
to rotate these antibiotics. So say patient had a good response to TMP-SMX; next choice when they get an infection
should be something new else they will develop resistance, so ampicillin or erythromycin. Macrolides
(azithromycin) and fluoroquinolones (levofloxacin) also good choices.
* Aminophylline is a phosphodiesterase inhibitor with minor bronchodilation effect that has some use in COPD.
Toxicities of aminophylline include tachycardia, tachyarrhythmias, tremors, and seizures. We have much better and
safer bronchodilators in asthma, namely the beta2 agonists like albuterol.
* Aminophylline used in COPD to increase central respiratory drive, improves diaphragmatic contractility, it makes
the work of breathing smoother.
* Say patient is long-time smoker and getting tachyarrhythmia. Next step is stop theophylline or aminophylline and
get an aminophylline level.
* Aminophylline is not cleared well in patients on erythromycin, ciprofloxacin, and cimetidine. Drugs that decrease
aminophylline levels include warfarin and phenytoin.
* Steroids should be monitors. Even with inhaled steroids patient can get cataracts, osteopenia, osteoporosis,
hypertension, fluid overload, diabetes, and so on. Demonstrate benefit (baseline PFTs) pre- and post-steroids.
* Most patients do not benefit from steroids if you do the measurements before and after a couple of weeks.
* Most important treatment for COPD is stopping smoking. Even stopping smoking after decades of damage has
benefit. Smoking cessation improves symptoms and mortality. Combine pharmacologic and counseling.
* When is the patient hypoxic enough to get chronic home O2? Criteria is PO2 < 55mmHg at rest. If PO2 between
55 and 60mmHg, likely you will be less than 90% oxygen sats. So in this group, look for end organ damage, so
erythrocytosis (Hct increased) as a compensatory mechanism or signs of cor pulmonale.
* So how much oxygen? It’s cheap, why not just give 100%? These patients have respiratory acidosis, 7.30/50/50,
so they have high PCO2, low PO2, “50/50 club.” Home O2 not only makes this patient feel better, it makes them
liver longer. We want this patient into the low 90% oxygen sat range. We don’t want to knock out their hypoxemic
drive to breath, else we’re in big trouble. This patient is already acidemic, so they need some hypoxia as a central
CNS respiratory drive. Otherwise they hypoventilate more, PCO2 goes up, pH goes down.
* Typical dose is home O2 1-2L via nasal cannula.

DO NOT DISTRIBUTE - 16 -
Study Notes – Internal Medicine James Lamberg 01Feb2010

* COPD patients should get pneumococcal vaccine every 5 years. They should get influenza virus vaccine annually.
They can also get the hemophilus influenza (faintly staining gram negative rod) vaccine once in a lifetime.
* What is the most important test for COPD? Answer is FEV1 as a measure of lung disease.
Bronchiectasis
* Destruction of the elasticity of the bronchial walls, leading to recurrent infections.
* Bronchiectasis seen in chronic bronchitis patients and in kids with recurrent infections (tram track sign on chest x-
ray), think about cystic fibrosis.
* Be aggressive with pulmonary physical therapy. Postural therapy, have them bend over, clap on their back to get
mucus moving. In cystic fibrosis, infection is likely to be pseudomonas aeruginosa.
* Patients may develop episodes of blood-tinged sputum or hemoptysis.
* Given a chest x-ray of a bronchiectasis patient where you see the liver impression on the right and gastric air-
bubble on the left. This is dextrocardia with situs inversus. Think Kartagener syndrome (primary ciliary
dyskineasia). This is immotile ciliary syndrome, not helping to brush out infection. Also have immotile sperm.
* Pulmonary toilet (pulmonary hygiene) is helpful here too, suctioning of the airways, chest physiotherapy, blow
bottles, and nasotracheal suction.
* These patients require the same vaccinations as in COPD, pneumococcal, influenza virus, hemophilus influenza.
Interstitial Fibrosis
* Almost always idiopathic. What PFTs would you expect? Pattern is similar to restrictive, all volumes are
diminished. So FEV1 and FVC diminished but ratio of FEV1/FVC is the same.
* These patients are generally not smokers. Chest x-ray will show increased white markings (from scars). If they are
a smoker, you would think emphysema.
* On exam you find pulmonary hypertension, cor pulmonale, finger clubbing (not specific, but implies chronic).
* Rarely will these patients respond to steroids. Measure pre-steroid PFTs and post-steroid PFTs. If no
improvement, then stop the steroids.
Sarcoidosis
* A 27yo woman comes to your office with painful erythematous papules that occurred one day before your visit.
She has no other complaints except joint swelling and pain that occurred three days ago. The physical exam reveals
low-grade fever, symmetric swelling of the knees, PIP joints, and MCP joints, with well demarcated 3-4cm papules
over the anterior aspects of the legs. The papules are red and tender. What is the next step in confirming diagnosis?
* The painful red nodules on the shins are erythema nodosum. Add in joint swelling and we could be talking about
Crohn disease. We don’t necessarily have information to know this is sarcoidosis. Generally no lung involvement in
Crohn disease.
* On chest x-ray with sarcoidosis, you see bilateral hilar lymphadenopathy. Other causes of bilateral hilar
adenopathy are granulomatous diseases (e.g. tuberculosis), bilateral lymphoma, metastatic adenocarcinoma.
* Sarcoid more commonly seen in younger patients and in African American patients.
* Sarcoid causes scaring, so you expect a restrictive lung pattern (decreased PFTs, normal FEV1/FVC).
* Sarcoid associated with Bell palsy, parotid gland enlargement, skin lesions (lupus pernio on face, erythema
nodosum on the lower extremities), uveitis (ocular emergency requiring steroids), bilateral hilar lymphadenopathy
leading to lung scaring and eventually honeycombing pattern, liver granulomas with elevated LFTs, CNS
involvement (treated with steroids), PFTs similar to restrictive disease.
* Uveitis usually occurs rather abruptly, pain in the eyes, photophobia. So either give steroids or refer immediately
to the ophthalmologist for slit lamp exam.
* Other disease involving the head that can cause blindness, seen in older patient, temporal arteritis.
* Unilateral hilar lymphadenopathy not seen in sarcoid. Worry more about things like TB, cancer, and metastasis.
* Hypercalcemia seen in sarcoidosis due to elevated levels of vitamin D so absorb more calcium, hypercalciuria too.
* Diagnosis is with tissue, so do a mediastinoscopy to biopsy nodes. Could biopsy supraclavicular node if it is
available. Tissue should show non-caseating granulomas.
* Kveim (Nickerson-Kveim, Kveim-Siltzbach) test is the equivalent of the PPD/mantoux test for TB. Kveim test is a
purified antigen from the granulomas of sarcoid patients. This is not a test of choice, want biopsy.
* Patients may have angiotensin converting enzyme (ACE) elevations, not sensitive or very specific.
Occupational Lung Disease
* Typically occurs 10-20 years down the road. Pneumoconiosis, asbestosis, silicosis, and coal worker’s lung.
* Smoking exacerbates and is superimposed on these diseases.
* All have a restrictive lung pattern so come with decreased PFTs.
* Pneumoconiosis seen in the mining industry or heavy metal exposure, gold, copper, silver, nickel, lead inhalation.
This causes a chronic inflammatory response, are ingested by macrophages, and causes scaring over the years.

DO NOT DISTRIBUTE - 17 -
Study Notes – Internal Medicine James Lamberg 01Feb2010

* Pneumoconiosis comes with opacities and irregular densities on CXR, “ground glass” appearance.
* No good treatment, remove patient from environment if still there, have patient quit smoking.
* In asbestosis there is a synergistic cancer effect with smoking.
* Asbestos found in older building insulation, pipes, boilers, old building construction, shipyard workers.
* No specific signs or symptoms, diagnosis found via occupational history.
* CXR shows calcification of the pleura (white areas). Primarily you see thick pleural bases, eggshell white.
* Mesothelioma is a specific and rare cancer seen with asbestosis, nearly always fatal. Detected by biopsy only.
* What is the most likely type of lung cancer a patient with asbestosis will get? Answer is adenocarcinoma or
squamous cell carcinoma primarily of the lung parenchyma, especially if a smoker. Answer is not mesothelioma.
* 70-fold increase in primary lung cancer in asbestos exposure with concurrent tobacco smoking.
* By the time you think to biopsy a cancer, it is likely too late. If you happen to get a biopsy, there will be barbell
shaped ferruginous body (asbestos fiber coated with iron-rich material).
* Treatment is to get patient to stop smoking.
* Silicosis is seen with rock mining, digging tunnels, digging quarries. Silica fibers are released from rock blasting.
* Restrictive disease. CXR nodules favor the upper lobes (similar to TB). These patients are at increased risk for TB.
* Patients with silicosis should get annual PPD testing.
* Coal miner’s lung associated with mining coal. Restrictive disease.
* Patients develop interstitial lung disease, small round densities typically mid to lower lobes.
* Encourage patient to quit smoking.
* Patients with coal miner’s lung who have joint complaints, such as symmetric swollen joints possibly with hand
involvement, you should think about rheumatoid arthritis (Kaplan syndrome) and check rheumatoid factor.
Pulmonary Embolism (PE)
* A 32yo woman is brought to the ED with acute onset shortness of breath and pleuritic chest pain (sharp pain,
worse with deep breath, coughing, or moving) that occurred while she was shopping. If you look down and see
answer choices like venous Doppler, ventilation perfusion scan, spiral CT scan, you know it is likely a question
about pulmonary embolus. This patient is not taking any medications except oral contraceptives. She has never been
sick before. Her respiratory rate is 26, is tachycardic, lungs are clear, rest of exam is normal. ABG shows mild
evidence hypoxemia. ABG 7.52/25/75. Drop PCO2 by 10, pH goes up by 0.08.
* If you see oral contraceptives (hypercoagulability due to estrogen) and sudden chest pain, thing PE.
* A-a gradient: 150 1.2*25 = 120. So 120 - 75 = 45 (normal < 15). So the problem is with gas exchange.
* A thrombus is a clot at its point of origin. An embolus is a broken off clot that traveled. A PE usually originates
from the deep veins of the legs or pelvis, thighs > calves.
* In a PE, the alveolus is just fine, the airway is just fine. But there is a clot blocking blood in the pulmonary artery.
So there is an area that is ventilated but not perfused, waste of ventilation, ventilation/perfusion (V/Q) mismatch.
* Physical exam will reveal an absence of problems, no wheezing, no rhonchi. So you need to do testing.
* To detect a V/Q mismatch, you order a V/Q scan. Patient breathes in nuclear-labeled air and an image is taken to
determine ventilation. Then IV nuclear material is labeled to RBCs and another image is taken to determine
perfusion. Normally there is a superimposed pattern of ventilation and perfusion. If there is a PE, you will see
ventilation but no perfusion in a particular area.
* V/Q scan is not gold standard. Results come as low, intermediate, or high probability of PE. The radiologist has a
large list of criteria to determine probabilities. With intermediate, you may be forced to do more tests.
* What is the first step in detecting a pulmonary embolus? Answer is V/Q scan (non invasive study).
* Another non-invasive test is a Doppler sonogram of the lower extremities.
* What is the quickest test to determine if a patient needs treatment for a PE? Answer is Doppler. If there is a blood
clot, that is enough to start with anticoagulation.
* Gold standard to determine if a patient has a DVT is a venogram. The test is painful; injection of dye into small
vein of the foot with imaging to see the dye. This is rarely necessary; only done if you have no choice.
* Gold standard to detect a PE is a pulmonary angiogram. Say the Doppler is equivocal and the V/Q scan is
equivocal, then you do the pulmonary angiogram.
* Pulmonary angiograms are used less widely now due to more acceptance of the spiral CT scan with contrast.
* A Doppler is a valid initial approach in a pregnant patient, since the presence of DVT warrants anticoagulation.
* Risk factors for pulmonary embolus (Wells criteria): immobilization (e.g. post-op, especially hip replacement),
cancer, HR > 100, PE is likely #1 diagnosis, clinical signs/symptoms of DVT, previous PE/DVT, and hemoptysis.
Other hypercoagulable criteria are not included in Wells, such as use of oral contraceptives.
* The DVT risk for hip surgery is so high that standard of care includes pre-op prophylactic anticoagulation
(subcutaneous heparin, 5000U subq q12h) as well as post-op anticoagulation.

DO NOT DISTRIBUTE - 18 -
Study Notes – Internal Medicine James Lamberg 01Feb2010

* Patients at risk for DVT get anticoagulated (subcutaneous heparin) or get sequential compression devices (SCDs).
* Patient may be ambulatory but have dementia so they don’t leave bed, use subcutaneous heparin.
* Patient is normally ambulatory busy businessman, takes long flight across seas and is suddenly short of breath, this
is most likely a PE case. Even sitting in a car for a long trip.
* Patients with CHF are at increased risk for DVT/PE due to poor cardiac output and blood stasis.
* Clots in strange places, like upper extremities (axillary vein) or in any artery, think hypercoagulable state.
* Factor V Leiden is the most common cause of thrombophilia. Factor V is necessary for protein C to work. Protein
C and S as well as antithrombin III (AT III) are necessary to prevent constant coagulation. Factor V Leiden causes
activated protein C resistance, a qualitative problem, so quantitative levels of protein C are normal.
* If the pulmonary embolism makes it peripheral to the pleura, the pleura can infarct and cause a pleural rub. You
could see this on CXR possibly. This is not the normal case; even with a massive PE the CXR will be normal.
* If a large pulmonary vessel is clotted, you can get signs of right ventricular strain, distended neck veins, still in the
presence of a clear lung exam. May see signs of cor pulmonale acutely, loud S2, P2 louder with inspiration, acute
right ventricular heave, right axis deviation on ECG, or ECG S1 Q3 T3 pattern (abnormal S wave in I, Q wave in III,
and flipped T wave in III, not specific or sensitive).
* When do you need to know for sure that this is a pulmonary embolus, so much so that you do the invasive and
possibly risk pulmonary angiogram or pulmonary venogram? You do it when patient is at very high risk of bleeding
if you anticoagulate them for treatment of the PE. Examples are recent major surgery, recent CVA, recent duodenal
bleeding ulcer, recent major trauma.
* A patient has a positive V/Q scan with symptoms. You do not think the patient is going to die from this clot. So
you want to prevent the clot from getting bigger. Treatment is anticoagulation. The fastest acting anticoagulation
agent is IV heparin, which will work within the hour. Keep patient on continuous heparin infusion. Heparin is not
thrombolytic therapy, so it will not break the clot. You expect the patient’s own endogenous system (protein C,
protein S, antithrombin III) to break up the clot and after time the vessel will recanalize. Heparin is given in bolus
(say 5000U) then infusion (say 10 or 15U/kg) at say 1000U/hour.
* Check heparin level with PTT (partial thromboplastin time) test. Aiming for time of 1.5-2.5 times the control. If
control in assay is 30 seconds, you want 45 to 75 seconds. If you’re at a PTT of 100, stop heparin for a few hours to
let the level come down as patient is at high risk of bleeding. If PTT low, increase heparin infusion rate.
* Treat patient with IV heparin for about 5 days. Start patient on oral anticoagulant (warfarin/coumadin) that they
will stay on for the next 3-6 months.
* Heparin works by potentiating antithrombin III effect.
* Warfarin works by inhibiting vitamin K dependent coagulation factors (II, VII, IX, X, plus protein C).
* Measure warfarin via PT (prothrombin time, “pro time”). Prothrombin no longer measured directly, it is done by
INR. Problem with PT test is control time varies too much. International normalized ratio (INR) range for a DVT is
between 2 and 3. If for stroke or prosthetic valve, INR between 3-4. So if INR is 1.7, increase oral warfarin dose to
get them above 2 for the DVT. If INR is 5, we need to reverse the effect of warfarin.
* Warfarin has a sizeable morbidity and mortality associated with it because there is variation in week-to-week or
even day-to-day dosing. So, we have to regularly check INRs.
* If patient bleeding with high INR, we give factors II, VII, IX, X, more specifically fresh frozen plasma (FFP).
* FFP comes from other human beings thus it can come with infections and transfusion reactions.
* If the patient has a high INR without bleeding, give vitamin K. Oral vitamin K will lower INR in 24-48 hours. IV
vitamin K will lower INR in 6-8 hours. Subcutaneous vitamin K not recommended.
* Drug interactions are important with warfarin, as many drugs affect the same P450.
* We do not start warfarin at the same time as we start the heparin. Warfarin inhibits protein C. Protein C has the
shortest half-life, shorter than factor VII. So the first effect warfarin has is to make the patient protein C deficient, so
they have more tendency toward hypercoagulability.
* So once PTT is 1.5 to 2.5 times control, then we can start the warfarin.
* 25yo woman stopped taking her birth control pills because she is scared of clots after reading about them on the
Internet. She has been gaining weight and stays at home most days. So, this patient got pregnant and now is
hypercoagulable due to estrogen, has a baby pressing on her pelvic veins, and is not moving around much.
Pregnancy increases risk for DVT. If this patient does get a DVT, then we can give heparin. Heparin is safe during
pregnancy. Warfarin is contraindicated in pregnancy, it is teratogenic. So we treat the patient with IV heparin for 5
days then send them home on subcutaneous heparin (10,000U subq TID as we need 20k to 30k units daily).
* Low molecular weight heparin (LMWH) has more consistent effect on PTT, so much so that we do not need to
follow repeated PTT levels. Once the patient has a stable PTT, we just follow the dose. LMWH is a once a day dose.
* The problem with LMWH is that they are very expensive, could be a few hundred dollars a day.

DO NOT DISTRIBUTE - 19 -
Study Notes – Internal Medicine James Lamberg 01Feb2010

* Say patient has life-threatening PE, like a saddle embolus at the main pulmonary artery. This causes a ventilation
perfusion mismatch at the entire lung. So even if you intubate the patient and deliver 100% O2 directly into the
trachea, no blood will be there to transfer the oxygen into the system.
* One sign this is a potentially fatal PE is that we are delivering 100% oxygen and the sats keep dropping, or blood
gas shows increasing hypoxemia. Another sign is acute right ventricular failure with hypotension. Hypotension in
the setting of pulmonary embolus is an ominous finding; the right ventricle is trying to pump against tremendous
resistance, so it will acutely fail, left atrium has no input thus no LV output.
* Treatment for life-threatening PE is thrombolytic therapy. Give tissue plasminogen activator (tPA) or
streptokinase (SK). This will lyse the clot, if there is enough time to save the patient.
* Why don’t we just give tPA in every patient? About 1% of patients getting thrombolytics die of a bleed. About
10% of patients getting thrombolytics have a major bleed.
* Say patient had major PE then became hypotensive and fell, resulting in an intracranial bleed. Or patient has major
PE and a recent GI bleed. Do you give tPA or streptokinase? Answer is emergently take patient to the OR or via
interventional radiology (IR) to do an embolectomy.
* We anticoagulate the embolus 3-6 months if there is a reversible cause. For instance, a post-op patient who got a
DVT or a patient who got a DVT after a plane ride. What about patients who have an on-going risk, like CHF or an
underlying coagulation disorder? Then we anticoagulate for as long as the risk exists. Say patient got a DVT in the
first month of pregnancy due to increased estrogen levels, then we anticoagulate for the entire pregnancy.
* What if the patient has a PE, is hemodynamically stable, but anticoagulation is contraindicated (e.g. recent major
hemorrhage). We have to protect the patient against future emboli, but can’t anticoagulate. So we put in an inferior
vena cava (IVC) filter. In less common instances, SVC filters can be placed as well. Some IVC filters are shaped
like an umbrella (IVC umbrella), some are disk shaped with pores, others may be mesh patterned.
* Say the patient has a PE, we diagnose and treat with the correct PTT then INR. What if the patient gets another
pulmonary embolism? Answer is place an IVC filter. The filter itself is thrombogenic, thus we do not stop the
anticoagulation if we do not have to.
Acute Respiratory Distress Syndrome (ARDS)
* 32yo man is admitted to the ICU with presumed gram-negative sepsis. He is placed on double gram-negative
antibiotic coverage and remains stable for the next 24 hours. Blood cultures grow pseudomonas sensitive to
ceftazidime and ciprofloxacin, which the patient has already been started on. The patient seems to improve over the
next day, until he suddenly develops severe dyspnea. Physical exam reveals diffuse crackles. An ABG shows
hypoxemia and hypercarbia. Diffuse alveolar densities are seen on CXR, compared with an unremarkable admission
chest x-ray. The lungs are flooded with fluid, so oxygen cannot diffuse through the fluid to the membrane.
* Premature newborns get similar problems (gas exchange problem at the alveolar-capillary membrane) when born
without enough surfactant.
* ARDS also known as non-cardiogenic pulmonary edema.
* Thus to prove ARDS, you have to rule out cardiac so normal pulmonary wedge pressure and LV end diastolic.
* ARDS occurs in gram-negative sepsis, burns, trauma with or without multiple PRBC transfusions, drowning,
narcotic overdose.
* Giving these patients oxygen does not do much to fix hypoxemia. Definition is PO2 < 60% on FIO2 > 60%, real
60% oxygen into trachea with patient intubated.
* Treatment is intubation and delivery 100% inspired oxygen. If an ABG shows PO2 low (like 45), then we add
PEEP (positive end expiratory pressure). PEEP holds the alveoli open, which helps to force the air through the fluid.
* Risk of giving PEEP is blowing a hole in the alveolus, barotrauma, causing pneumothorax.
* If you cause a pneumothorax and do not recognize it, the PEEP will continue to push air out, leading to a tension
pneumothorax. Patient will become hypotensive, mediastinum shifts away.
* As intrathoracic pressure gets increased with PEEP, you increase right-sided venous return at the expense of the
left ventricle; so think of it as compressing the left ventricle, resulting in hypotension.
* Patient on ventilator, CXR normal, and patient getting hypotensive. Answer for next step is to decrease PEEP.
* Oxygen toxicity can occur with FIO2 > 50%, which causes fibrosis.
* To measure capillary wedge pressure (PCWP), insert a pulmonary artery catheter (Swan-Ganz catheter). Insert
into a large vein, like internal jugular or subclavian (IJ safer because further from lung). Catheter placed through
RA, RV, pulmonic valve, into outflow tract with balloon deflated. Inflating the balloon too much could rupture the
pulmonary artery. On the catheter is a pressure transducer, so we can measure the pressure in the pulmonary artery.
* At the end of diastole, the ventricle is maximally full, aortic valve is closed, so this is the left ventricular end
diastolic pressure; the mitral valve is still open so the pressure is transmitted back to the left atrium, that pressure
transmits back to the pulmonary veins, and back to the pulmonary capillary.

DO NOT DISTRIBUTE - 20 -
Study Notes – Internal Medicine James Lamberg 01Feb2010

* So we inflate the balloon, it travels down and gets wedged in the pulmonary capillary. We get a continuous
column of pressure at that capillary; which is in continuous communication with the left ventricle.
* Say LVEDP normally is 7-12mmHg, what if the pressure is 5mmHg? The low pressure suggests it is non-
cardiogenic pulmonary edema (ARDS). So pulmonary edema with low LVED pressures suggests ARDS.
* Say pulmonary capillary wedge pressure (PCWP) is 25mmHg with edema? That is cardiogenic.
* You can also measure cardiac output via the Fick equation with a Swan-Ganz catheter.
* Catheter has a thermometer on the tip along with balloon and pressure transducer. Thermometer tells you what the
temperature of the blood is. Take chilled saline that you know the temperature and fluid amount. You inject the
saline and the catheter measures continuous temperature. The quickness and degree to which the temperature falls in
response to the saline helps determine the rate of flow and thus the cardiac output (L/min depending on BSA).
* Cardiac index should be > 2.2 L/min/BSA. Below 2.2 is very poor flow.
* Cardiac index may be normal or high in ARDS, but in cardiogenic pulmonary edema it is low.
* You can follow cardiac index to help determine how much diuretic or inotrope to give.
Sleep Apnea
* Sleep apnea is cessation of breathing during nighttime sleep, occurring at least 10-15 times per hour and at least 10
seconds per episode.
* Patient does not realize this is happening. So either the patient has daytime somnolence (fall asleep at my desk,
drink more and more coffee and still falling asleep, falling asleep while driving) or spouse notices loud snoring.
* Major cause of sleep apnea is obstruction of the upper airway at the larynx and epiglottis.
* Seen in obese patients. Will have tremendous intolerable (to others) snoring.
* About 5% of cases are central apnea where the issue is not in the upper airway but in the respiratory drive.
* Treatment is with CPAP (continuous positive airway pressure) mask at night, similar concept as PEEP.
* The positive pressure in CPAP is being delivered to the mouth, so no risk of barotrauma like in PEEP.
* Testing done in sleep lab, patient hooked up to EEG, ECG, pulse ox, respiratory rates measured. It is not a popular
test because the patient is trying to fall asleep while hooked up to all these devices.
* More invasive treatment if CPAP fails is surgical intervention to open the airway; not always successful.
Lung Cancer
* Most common cause of cancer death in both men and women is lung cancer.
* Women: 2nd cause is breast cancer (1st if non smoker), 3rd is colorectal cancer.
* Men: 2nd cause is prostate (1st if non smoker), 3rd is colorectal.
* A non-smoker that gets lung cancer is likely adenocarcinoma.
* Most common initial presentation is a cough. This is a very non-specific symptom; smokers usually cough. So,
look for a change in the cough. Maybe they have superimposed bronchitis. This is a difficult diagnosis at times.
Taking all lung cancers that present to the physician’s office, 5-year survival rate is about 5%.
* We have no good diagnostic test to improve survival in smokers who develop lung cancer.
* Cough with another symptom, like pleuritic chest pain, blood-tinged cough, post-obstructive pneumonia.
* Once a patient has about 10% weight loss with a new lung cancer diagnosis, nearly all are unresectable.
* Post-obstructive pneumonia is when patient has recurrent pneumonia in the same location. Patient gets pneumonia,
gets better, CXR 3 weeks later still shows a patchy spot. That is because secretions cannot clear due to obstruction.
* So any smoker with pneumonia you must follow CXR until it is clear. If not cleared, must get bronchoscope.
* Chronic smokers generally develop chronic hoarseness/laryngitis. If a patient suddenly develops hoarseness, you
should worry that metastasis has traveled to the recurrent laryngeal nerve (outside lung). Testing is with
laryngoscopy and patient phonating to determine if a vocal cord is paralyzed (ENT consult).
* Superior vena cava syndrome implies a big mass in the chest is impinging the SVC, this implies unresectable
cancer. Patient will get swelling in face with redness, called facial plethora (Pemberton sign).
* If cancer is found and centrally located, you can do a bronchoscopy and biopsy.
* If the lesion is peripherally, one can do a CT guided needle biopsy.
* If the lesion is somewhere in the middle, must do an open biopsy (can be done laparoscopically these days).
* Squamous cell carcinoma is generally centrally located, so more likely to be bronchoscopy with biopsy.
Associated with paraneoplastic syndrome of hypercalcemia due to parathyroid hormone like substance secretion.
* Small cell (oat cell) carcinoma generally centrally located, associated with SIADH (hyponatremia with
euvolemia), Eaton-Lambert syndrome, early SVC syndrome.
* Eaton-Lambert syndrome is neuromuscular end-plate injury, differentiated from myasthenia in that muscles
improve with use (MG patients worsen with use).
* Adenocarcinoma carcinoma and large cell carcinoma tend to be more peripheral, so needle aspiration biopsy or
thoracotomy to get tissue for diagnosis.

DO NOT DISTRIBUTE - 21 -
Study Notes – Internal Medicine James Lamberg 01Feb2010

* Non-small cell cancers can only be cured if they are resected prior to metastasis. To determine if the patient can be
cured, look for lack of metastasis. Signs of metastasis would be malignant pleural effusion, brain involvement, bone
involvement, local contiguous structures (e.g. liver, esophagus), local extension into nearby organ.
* You can have local nodes and be cured with resection only if the nodes are on the same side of the cancer.
* Even if you have a tiny lung cancer and a single node on the contralateral mediastinum, it’s too late to cure.
* Patients may still be resected for palliative measures, such as a large tumor causing obstruction.
* Non-small cell lung cancer can be palliated with chemo and radiation, but this is not a cure.
* Small cell cancer metastasizes early. So by the time you make the diagnosis, you must assume there has been
micro metastasis and thus resection is not an option for cure.
* One could be cured with chemotherapy in small cell cancer, but 95% are not cured at 6 months and die.
* The only good news about lung cancer is the rates are decreasing as people are smoking less and less.
* The greatest impact to reduce lung cancer rates is through education of teenagers before they start smoking.

Kaplan Videos – Nephrology with Dr. Barry Weiss, MD


Questions should be core, straightforward topics with obvious answers if you know the content. The questions are
not meant to trick the test taker. These are classic cases, classic scenarios.
Acute Renal Failure
* If you’re told a patient has bad breath, you can be sure we’re talking about Zenker diverticulum. A canoe trip in
Connecticut, we’re talking about Lyme disease. There are similar key words and patterns to know for nephrology.
* 62yo man comes to the ED for nausea and vomiting. Lab tests show creatinine of 3.6, it was normal two months
ago. Why are the kidneys failing?
* Three main causes of acute renal failure: pre-renal azotemia (non kidney problems, inflow issues like circulation),
intra-renal azotemia (intrinsic disease), and post-renal azotemia (outflow).
* Pre-renal azotemia is due to poor perfusion of the kidney. Causes include volume depletion (dehydration, burns,
third-spacing into abdomen from pancreatitis), cardiovascular (hypotension, poor cardiac output especially CHF),
decreased oncotic pressure (low albumin edematous states like cirrhosis, nephrotic syndrome), renal artery
vasoconstriction (meds like NSAID, ACE-I, and anatomic like renal artery stenosis).
* Some of the blood flow into the kidney is prostaglandin-dependent and NSAIDs block prostaglandin.
* Pre-renal azotemia is important because it represents low blood flow to the kidneys; best considered as transient
ischemia. If untreated, can progress to renal damage (acute tubular necrosis) and intrinsic renal failure.
* Must be distinguished from intra-renal and post-renal azotemia because the treatments are very different.
* Post-renal azotemia causes include bladder outlet obstruction (prostate enlargement, pelvic tumors, urethral
stricture), ureteral obstruction (tumors, stones, papillary necrosis). Significance is back pressure damages kidneys.
* Intra-renal failure is damage to the kidney itself, also called intrinsic renal disease or acute tubular necrosis (ATN).
* Tests for post-renal azotemia include prostate exam, catheterization or bladder scan (to detect post-void residual
urine volume), best test is ultrasound of the kidneys, ureters, and bladder (U/S KUB).
* You expect about 50mL of urine left in bladder after urination, so if a post-void scan shows 300mL of urine, you
should worry about outlet obstruction.
* Normal BUN/Creatinine ration is 10 to 20. Post-renal could also have a BUN/Cr of 10-20. < 10 is intra-renal.
* Tests for pre-renal azotemia include high BUN/Cr ratio (> 20) is the most important. In low flow states, functional
kidneys retain sodium to maintain vascular volume, so low urine Na+ (< 10-20) and low fractional excretion of Na+,
or FENa, (< 1%).
* What happens is the kidneys don’t get enough blood, they think you are bleeding to death (evolutionary response),
so they retain lots of sodium to help build up intravascular volume, thus urine sodium is very low.
* In low-flow states, functional kidneys will concentrate urine to retain fluid in the body, so high (concentrated)
urine specific gravity and high (concentrated) urine osmolality.
* 62yo man comes to the ED with nausea and vomiting, creatinine was normal now is 2.6. Has orthostatic
hypotension. BUN 82, urine Na+ 6, urine specific gravity 1.028. Answer to why they are failing is pre-renal
azotemia. BUN/Cr is high, urine sodium is low so kidneys are retaining it.
* Treatment for pre-renal azotemia is fluids.
* Intra-renal azotemia: BUN/Cr is < 10-15. Urine sodium >20-40 because kidneys are failing so they lose the ability
to protect sodium, it just leaks out. Can’t concentrate urine, so urine specific gravity < 1.015, urine osmolarity <
350. Also look for casts, granular and RBC. Casts come from the kidney.
* If patient on diuretics, like furosemide, then that may cause high urine sodium do don’t assume intra-renal failure.
* So in pre-renal BUN/Cr is high with low urine sodium, in intra-renal BUN/Cr is low with high urine sodium.
* In summary, you’re faced with a high creatinine level that is new (azotemia). Follow azotemia algorithm.

DO NOT DISTRIBUTE - 22 -
Study Notes – Internal Medicine James Lamberg 01Feb2010

* Algorithm: First thing to do is exclude pre-renal azotemia by looking for lack of BUN/Cr > 20, urine Na < 10,
FENa < 1%, Osm > 500, high specific gravity. If you find pre-renal, you treat the cause (most causes you give them
fluid, or treat CHF or renal artery stenosis). Next thing to do is exclude post-renal azotemia by sonogram and/or
catheter looking for lack of obstruction. If you find post-renal, treat it. So, if you don’t see pre-renal or post-renal
problems, it is most likely intrinsic renal. Signs of intra-renal disease are BUN/Cr < 10, urine Na > 20, FENa > 1%,
Osm < 200, low specific gravity.
Intra-Renal Disease: Tubulo-Interstitial Disorders
* Acute tubular necrosis (ATN) occurs in prolonged ischemia (hypotension, prolonged pre-renal azotemia, arterial
insufficiency/occlusion, previously called “shock kidney”), toxins (radiologic contract, drugs like aminoglycosides
and amphotericin B, pigments like myoglobin).
* Elderly man dropped pen under bed and went to go retrieve it. Once uncer the bed he couldn’t get out. Neighbor
realizes man hasn’t come out of the house for a few days and finds the man stuck under the bed. Patient has been
lying on the big muscles of the legs for a long time with no good circulation. The broken down muscle tissue
releases myoglobin, which goes through the renal tubules and is toxic to the tubules.
* Phases of ATN are prodromal, oliguric (when kidney starts showing changes), and post-oliguric (diuretic).
* Patients who have diabetes (and are taking metformin) and get contrast dye should have the metformin stopped
prior to the procedure.
* Allergic interstitial nephritis is an allergy reaction to a drug. Typical drugs are beta-lactams (e.g. cephalosporins,
penicillins), sulfonamides, rifampin, allopurinol. Presentation is fever, rash, peripheral eosinophilia. Key findings
are urine eosinophils. If you see urine eosinophils, think allergic interstitial nephritis and look for what drug needs to
be stopped, most commonly a cephalosporin. Treatment is stop drug and short-term steroids to cool down reaction.
* Deposition disease causes include hemoglobin (hemolysis), myoglobin (rhabdomyolysis), protein (multiple
myeloma with Bence Jones protein), oxalate crystals (ethylene glycol, vitamin C), uric acid crystals (chemotherapy,
tumor-lysis, gout), hypercalcemia (hyperparathyroidism).
* Infection causes include pyelonephritis with WBC casts and bacteria in urine.
* Drugs and toxin causes include analgesics (NSAIDs), antibiotics (aminoglycosides, amphotericin B,
cephalosporins, expired tetracyclines leading to Fanconi syndrome), cancer drugs (cisplatin, methotrexate,
cyclosporin, mitomycin C), radiation nephritis, heavy metal poisoning (lead, mercury, gold, lithium).
Intra-Renal Disease: Glomerular Disorders
* Vasculitis causes include Wegener granulomatosis (lesions in lung, nasal, sinus, and ANCA anti-neutrophil
cytoplasmic antibodies), Henoch-Schonlein purpura, polyarteritis nodosa, thrombotic thrombocytopenic purpura,
cryoglobulin disease, hypertension, diabetes.
* Patient has lung lesions and recurrent sinus infections. Also has kidney issues. Think Wegner and order ANCA.
* Glomerulonephritis causes include post-streptococcal (do ASO titer, follows strep skin or throat infection), Berger
disease (IgA mesangial deposition after viral illness, young children), collagen vascular disease (e.g. SLE),
Goodpasteur syndrome (anti-GBM antibodies in lung and kidney), Alport syndrome (renal failure and deftness).
* Teenager presents with intrinsic renal disease. Had a sore throat a week before. Answer is order ASO titer.
Glomerulonephritis
* Intra-renal disease has high urine sodium (> 20) and high FENa (> 1%), abnormal urinary sediments (hematuria,
RBC casts). Renal biopsy is the most definitive test; not done often in clinical practice because of risk.
* Nephrotic syndrome occurs in vasculitis and glomerulonephritis. Massive proteinuria > 3.5g/day, this is 4+
dipstick positive with grams on measurement. These patients will lose their oncotic pressure and develop edema.
They can run their total albumin way down and get anasarca (“marshmallow man” appearance). Hyperlipidemia also
seen. Best test to establish a diagnosis of nephrotic syndrome is 24-hour urine protein.
* So you have a patient with azotemia. You rule-out pre-renal and post-renal. You see blood in the urine and RBC
casts. Dipstick is positive for protein and patient has edema. How do you prove it is nephrotic? 24h urine protein.
* Best test to determine the type of glomerulonephritis causing the nephrotic syndrome is renal biopsy.
* Best initial test to distinguish causes of intrinsic renal disease is a urinalysis.
* Urinalysis can show infection (WBC, bacteria), drug hypersensitivity (eosinophils), glomerulonephritis (RBC
casts), nephrotic syndrome (lots of protein), myeloma (Bence Jones), uric acid (gout), myoglobinuria (blood on
dipstick due to myoglobin, no RBCs, high creatine kinase CK in blood).
* 62yo man comes to the ED for nausea and vomiting, creatinine 3.6 (new), no medications. Exam is normal except
large prostate. BP 11/70, pulse 110, BUN 72. Urine sg 1.023. Urine Na+ 9. What is the best initial treatment? You
may be tempted to pick sonogram due to outlet obstruction, but a 62yo man probably has a large prostate due to
BPH. If you look at the BUN/Cr, it’s 20 plus concentrated urine. So pre-renal azotemia. Patient has high pulse and
nausea with vomiting, so likely dehydrated. Best initial treatment is fluids.

DO NOT DISTRIBUTE - 23 -
Study Notes – Internal Medicine James Lamberg 01Feb2010

* 62yo man comes to the ED with nausea and vomiting. Creatinine is 3.6 (new) and is on NSAIDs. Exam normal
except large prostate. BP 110/70, pulse 110, BUN 41, urine sg 1.004, RBCs, no casts. Urine Na+ 42. BUN/Cr is
close to 10, can’t dilute urine even though he is vomiting/dehydrated. This is likely intrinsic kidney (intra-renal)
azotemia. Could have hypoperfusion due to NSAIDs. Could have BPH or cancer causing large prostate obstruction.
Could have bladder cancer or kidney stones due to RBCs in urine. What is the next step though? Answer is
sonogram to exclude post-renal azotemia. Once that is out of the picture, we assume renal disease and follow that.
So we ruled out pre-renal with urine originally, but still need to rule out post-renal.
* 14yo boy comes to see you with swollen hands and legs. He has a recent sore throat, afebrile, BP 140/90 (pretty
high), exam confirms edema. Urine shows proteinuria and RBC casts. This is post-streptococcal glomerulonephritis.
What blood test would be most helpful for confirming diagnosis? Answer is ASO titer.
End Stage Renal Disease (ESRD)
* ESRD is renal failure that is incompatible with survival. In order to survive, you need dialysis or a transplant.
* Most common causes are diabetes (30-50%), hypertension (10-25%), glomerulonephritis (15%), polycystic kidney
disease (5-10%), and unknown (10-15%). Most common cause of dialysis is diabetic kidney disease.
* Need for dialysis is not determined by creatinine level. It is determined by the presence of acute manifestations.
* Chronic manifestations of ESRD occur whether or not you are on dialysis. Hypocalcemia with osteopenia occurs.
There is a lack of 1,25-dihydroxy vitamin D, which is involved in calcium reabsorption in the GI tract. This causes
secondary hypoparathyroidism, calcium is leached from bone causing osteopenia. Hypermagnesemia occurs due to
failure to excrete magnesium, usually asymptomatic. Accelerated hypertension (renin-angiotensin action) occurs and
is a big deal because it may be hard to control blood pressures; the combination of accelerated hypertension and
hyperlipidemia leads to accelerated atherosclerosis leading to cardiovascular disease. Anemia occurs due to lack of
erythropoietin production in the kidneys, start to occur when the creatinine gets to about 3. Immunosuppression
occurs so increased infections. Pruritus occurs presumably due to toxins circulating in the blood that are normally
excreted by the kidney.
* Acute manifestations of ESRD include hyperkalemia (look for peaked T-waves on ECG), the mnemonic used is
“the T-waves are so sharp and pointed that you don’t want to sit on them” meaning do not sit and wait before
treating. Metabolic acidosis occurs but kidneys cannot compensate, so must get dialysis. Fluid overload occurs to the
point of pulmonary edema and the kidneys cannot remove the fluid, so need dialysis. Pericarditis occurs due to
toxins causing inflammation of the pericardium and build-up in the pericardial sac of fluid, needs dialysis.
Encephalopathy occurs at to high creatinine levels, and can lead to them being obtunded and even comatose, needs
dialysis. All acute manifestations are indications for dialysis.
* Dialysis can be done via blood (hemodialysis via a machine) or peritoneal dialysis (fluid instilled into peritoneum
then removed).
* 62yo dialysis patient returns from a 4-day fishing trip and feels poorly. This is a clue up front because most people
on dialysis get it every 2-3 days. Patient has nausea, vomiting, and shortness of breath. ECG shows tall peaked T
waves. Cardiac exam shows peripheral edema and signs of CHF. Creatinine is 5.2, bicarb is 16 (metabolic acidosis).
What is the best treatment for this patient? Answer is dialysis now, not furosemide for CHF, not kayexalate for
potassium, not bicarbonate for acidosis. This is an indication for dialysis, not to mess around with other treatment.
* Some key words in renal disease to remember. Eosinophils in urine, think drug reaction like cephalosporins. RBC
casts, think glomerulonephritis. Heavy proteinuria, think nephrotic syndrome. High triglycerides think nephrotic
syndrome. Hemoptysis, think Goodpasteur. Radiologic contrast, think acute tubular necrosis (ATN). Urine Na+ <
10, think pre-renal and give fluids. BUN/Cr > 20x normal, think pre-renal give fluids.
Electrolyte Disorders: Hyponatremia
* Hyponatremia is arguably the most common of the sodium and potassium abnormalities seen in the hospital.
* Hyponatremia mild or gradual change the patient will most likely be asymptomatic. With moderate change,
symptoms include headache and confusion. With severe or rapid change, symptoms include seizures, coma.
Emergency treatment is hypertonic saline (3% or 5%).
* Causes of hyponatremia include pseudo-hyponatremia, where sodium level measures low but artifacts from other
substances make it appear low, such as in high osmolality of hyperglycemia (1.6mg% sodium per 100mg% glucose).
So in hyperglycemia, the patient’s sodium is not truly low thus do not treat the sodium, treat the glucose.
* Pseudo-hyponatremia can occur also with normal osmolality, lab artifact from hyper-trigleridemia or hyper-
proteinemia (e.g. multiple myeloma). Not likely to show on the exam but can happen in clinical practice is when a
phlebotomist draws blood at the antecubital fossa when D5W is running at a distal hand IV site, showing
hyponatremia (diluted) and hyperglycemia (from the dextrose).
* “Real” hyponatremia is lab hyponatremia (from a basic metabolic panel) with low osmolality.

DO NOT DISTRIBUTE - 24 -
Study Notes – Internal Medicine James Lamberg 01Feb2010

* Hyponatremia with high osmolality (hyperglycemia), hyponatremia with normal osmolality (high lipids or
protein), and hyponatremia with low osmolality (true hypoglycemia, multiple causes).
* First step in determining why a patient is hyponatremic is determining the osmolality. Osmolality can be
determined by direct measurement (best) or by calculation (osmolality = 2*Na + glucose/18 + BUN/2.8, or a simpler
but less perfect method is 2*Na + 10). Normal blood osmolality is between 280 and 300.
* Hyponatremia with high osmolality caused by increase solute in the blood, usually glucose. Causes osmotic fluid
shift from intracellular space into the blood. Fluid shifted into blood dilutes Na+ level. There is a magnitude effect,
where sodium falls 1.6 per 100mg/dL increase in glucose. Say glucose is 342 and sodium is 126; that glucose
accounts for about 3.2 in sodium drop, so sodium is actually 129 adjusted. So the glucose does not explain all of the
drop in the sodium.
* Patient presents with diabetes. Labs show glucose of 402, sodium of 131. Does the glucose explain the level of
hyponatremia? Here, the glucose is about 300mg/dL high, so 3*1.6 is 4.8 difference for sodium. Thus adjusted
sodium is 136. Thus sodium is normal so the glucose explains this pseudo-hyponatremia.
* With normal osmolality pseudo-hyponatremia, in the healthy patient there is a small amount of protein/lipid per
block of blood, with a normal amount of sodium. When protein/lipid increase, there is less space for the sodium and
water component of blood. So per unit of blood, there is less sodium even though the concentration has not changed.
* Patient presents with weakness, sodium is 128, glucose is 142. Serum osm is 292 (normal). Total protein is 7.2
(normal). What test will show the cause of the patient’s hyponatremia? Answer is serum lipids (triglycerides). We
have low sodium with normal osmolality, hyper-proteinemia was ruled out. So hyper-triglyceridemia is next.
* Hyponatremia algorithm: once we rule out high osmolality and normal osmolality, lab shows osmolality is low. In
this case, the next step is to measure the extracellular fluid volume, so the patient’s volume status. High volume
status assessed with edema, low volume status assessed with tachycardia and orthostatic hypotension.
* If the ECF is high (hypervolemia), the patient is fluid overloaded and the cause is almost always an edematous
condition, including cirrhosis and congestive heart failure. If ECF is low (hypovolemia), the patient has fluid loss
with causes including GI, kidney, diuretics, sweat, and burns. If the ECF is normal (euvolemia), there are multiple
causes including SIADH.
* Low osmolality with hypervolemia, edematous conditions. Sodium is gained, accompanied and exceeded by
secondary water gain. Diagnoses to consider are congestive heart failure, nephrotic syndrome, and hepatic failure or
cirrhosis.
* Low osmolality with hypovolemia, volume depletion. Extrarenal loss (urine Na+ < 10) due to GI loss from
vomiting, diarrhea, tube drainage, bowel obstruction, or due to skin loss from burns, sweat. Renal loss (urine Na+ >
10) due to diuretics, salt-wasting nephropathy (renal failure), hypoaldosteronism, glucocorticoid deficiency.
* Low osmolality with euvolemia, normal volume states. SIADH, water intoxication (> 12-20L/day, also known as
psychogenic polydipsia), and hypothyroidism.
* SIADH has urine osmolality inappropriately high in the face of hyponatremia with low serum osmolality. Normal
renal, adrenal, thyroid function. May be a drug side effect. So again, ADH is secreated causing the patient to retain
fluid and excrete concentrated urine. Blood osm is low/dilute, urine osm is high/concentrated.
* Causes of SIADH include lung disease, brain disease, neoplasm-related (most common is small/oat cell in the
lung), drug side effect.
*Drugs that increase ADH secretion include TCAs, MAO inhibitors, fluoxetine, carbamazepine, narcotics,
phenothiazines, clofibrate, vincristine, cyclophosphamide, vinblastine.
* Drugs that increase ADH action include chlorpropamide, tolbutamide, theophylline, NSAIDs, carbamazepine.
* Drugs that mimic ADH function include oxytocin, used in labor induction.
* Drugs to pay attention to here include TCAs, phenothiazines, vincristine, cyclophosphamide, chlorpropamide.
* Diagnosis of SIADH is accomplished by seeing hyponatremia, low serum osm, high urine osm.
* Treatment is to reverse the cause if possible (e.g. fix pneumonia, stop drug), restrict fluids, furosemide with
normal saline (to raise blood sodium), and if severe and can’t find a reversible cause then demeclocycline or lithium.
* Emergency treatment for serious hyponatremia is hypertonic saline.
* 82yo man presents with 1 week of vomiting and 2 days of confusion. Takes thyroxin, fluoxetine, and
chlorpropamide. Sodium is 126, glucose 342, and has pneumonia. What are the possible causes of hyponatremia?
Vomiting could cause dehydration. Hypothyroid can cause ADH syndrome. Fluoxetine and chlorpropamide can
cause SIADH. High glucose could cause pseudo-hyponatremia. How do you determine the cause? You need
osmolality level and volume status. You also need urine sodium if volume status is low. Get a urine osmolality if
volume status is normal.
* Key points about hyponatremia: determine serum osmolality, high and normal osmolality are artifacts of high
glucose/lipids/protein, with low osmolality you need to determine ECF.

DO NOT DISTRIBUTE - 25 -
Study Notes – Internal Medicine James Lamberg 01Feb2010

* Correcting hyponatremia too quickly can cause central pontine myelinolysis, resulting in quadriplegia. The more
appropriate term now used is osmotic myelinolysis.
Electrolyte Disorders: Hypernatremia
* Hypernatremia is likely the least common of the sodium and potassium abnormalities.
* Causes include fluid loss in excess of Na+ loss commonly occurs in insensible loss from sweat (extreme exercise),
burns, and commonly occurs in GI loss from diarrhea (e.g. children), and less commonly in diabetes insipidus (DI)
where water loss is up to 20L per day. DI can occur due to central causes (idiopathic, tumor) and nephrogenic causes
(lithium, various renal diseases).
* What is the most common cause of hypernatremia? Answer is dehydration.
* Symptoms include lethargy, confusion, seizures, and coma.
* Treatment for dehydration is isotonic fluids with slow correction of sodium (1mEq/2h). If you correct the sodium
too rapidly, it can cause seizures due to cerebral edema.
* If you cannot correct the sodium, think about diabetes insipidus.
* If you think the patient has DI, do a water restriction test. In a normal person, the urine volume will decrease with
time and the urine output will be more and more concentrated. In diabetes insipidus, there is no change in urine
volume because there is no ADH to do it. The urine will not change concentration either. Then, the patient is given
ADH. If there is no response to the ADH, the problem is the kidneys (nephrogenic DI). If there is a response to the
ADH, meaning urine output drops and becomes more concentrated, then the problem is in the brain (central DI).
* Treatment of central diabetes insipidus is to correct the underlying problem if possible (e.g. lithium, brain tumor),
give vasopressin in the form of desmopressin (DDAVP) usually intranasal or sometimes subcutaneous or IV.
* Treatment of nephrogenic diabetes insipidus is to give thiazides or NSAIDs. Thiazides cause Na+ depletion and
water resorption in the kidney. NSAIDS increase prostaglandin (PG1), which impairs urinary concentration ability.
Electrolyte Disorders: Hypokalemia
* Hypokalemia common causes are GI loss from diarrhea, tube drainage, also medication such as diuretics causing
renal loss, beta-agonists (e.g. albuterol) that increase potassium entry into cells, and insulin that increases potassium
entry into cells.
* Rare causes of hypokalemia include hyperaldosteronism, like Conn syndrome primary hyperaldosteronism from
renal disorder, or Cushing syndrome from adrenal excess, or Bartter syndrome from volume depletion secondary to
impaired renal resorption of NaCl. Other causes include licorice in high quantities due to mineralocorticoid effect of
glycyrrhetinic acid.
* 73yo man with COPD and CHF has had diarrhea for 3 days. Now has increased SOB (shortness of breath). Meds
include furosemide and albuterol. Received multiple albuterol treatments in the ED (emergency department).
Identify three possible causes of hypokalemia. Causes include diarrhea with loss of potassium in the stool, the
furosemide with loss of potassium in the urine, and the albuterol with increased cellular resorption of potassium.
* Symptoms of hypokalemia if mild are asymptomatic, moderate include muscle weakness, muscle cramps, and
non-specific T and U wave changes. With severe hypokalemia, cardiac arrhythmias occur. At any level of
hypokalemia, digitalis/digoxin toxicity is potentiated.
* Treatment of hypokalemia is done by replacing potassium. Whenever possible, replace orally. IV replacement
maximum is 10mEq/hour via peripheral IV or 20mEq/hour via central line. An excessive rate causes cardiac
conduction defects including heart block and/or asystole. Potassium is one of the chemicals used for lethal injection
because it causes heart block and asystole.
Electrolyte Disorders: Hyperkalemia
* Causes include increased potassium intake, such as an error in the hospital with excessive potassium
administration. This is uncommon except in conjunction with certain medications (e.g. potassium sparing diuretics,
ACE-inhibitors). Pseudo-hyperkalemia can occur as an artifact from venipuncture or cell lysis from shaking the
blood tubes. Hemolysis of cells causes the potassium to be released from the cells, leading to falsely high potassium
levels. This can also occur if blood is drawn proximal to a running IV drip that contains potassium. Movement of
potassium from cells into the blood can cause hyperkalemia, occurs in acidosis (H+ into cells, K+ out),
rhabdomyolysis, and rarely familial periodic paralysis. Another causes is deceased renal excretion from renal failure
or medications like spironolactone and ACE-Is. Hypoaldosteronism and adrenal insufficiency are other cause.
* Symptoms include weakness that increases with potassium level, sudden cardiac conduction defects preceded by
classic ECG changes (talk pointed peaked T-waves). Patients needs treatment if ECG changes present.
* 67yo woman with diabetes and hypertension is on glyburide and ACE inhibitor. She presents for a routine
checkup. Labs reveal creatinine increased to 4.5 and potassium is 6.6. Why did her potassium become high? There
are a number of reasons include the ACE-I, could be renal failure. How will you treat this?

DO NOT DISTRIBUTE - 26 -
Study Notes – Internal Medicine James Lamberg 01Feb2010

* Treatment should start with protection of the heart. Give CaCl or Ca++ gluconate infusion. Next step is to lower
the potassium level by driving potassium into the cells. This is done with glucose and insulin, also with bicarbonate
as the alkalosis drives potassium into cells. They still have potassium in their body but it is no longer in the
circulation where it would induce cardiac arrhythmia. Last step is to get rid of the potassium from the body. This is
done with cation-exchange resin (kayexalate) or dialysis if renal failure.
Acid-Base Disorders
* A normal blood pH is 7.4 +/- 0.03. A pH < 7.37 is acidosis and pH > 7.43 is alkalosis.
* Arterial blood gas (ABG) gives you pH, where you can determine if this is acidosis or alkalosis. You get PCO2
(normal 35-45) and should determine respiratory rate/volume, these changes are rapid. Hyperventilation lowers
PCO2 and breath holding or shallow respirations will increase PCO2. You get HCO3 (normal 20-28), with
bicarbonate working as a metabolic buffer, these changes are slower (2-5 days). ABG also gives O2 and PO2, which
can vary by the altitude you live at. O2 sat typically in mid-90s, should be above 90% normally.
* Respiratory alkalosis is caused by hyperventilation, anxiety (acute pain, pain attack), acute pulmonary disorder
(pulmonary embolism, pneumonia), early aspirin toxicity. Drives down the PCO2 with no time for metabolic
compensation.
* Respiratory alkalosis will have increased pH, decreased PCO2, HCO3 unchanged because not enough time.
* 24yo female comes to the ED feeling like she is suffocating, has tightness in her throat and chest. She appears
anxious, RR is 36. ABG shows 7.52/25/PO2/27/99%. PCO2 at 25 is low. This is respiratory alkalosis, likely due to
the hyperventilation.
* What if the patient above had an O2 sat of 76%? This is not acute anxiety or panic attack. This is a lung problem.
Could be a patient on birth control pills with acute pulmonary embolism. Either way it is a lung problem.
* Metabolic alkalosis is either a loss of H+ or a gain of HCO3. Common causes of H+ loss include prolonged
vomiting or other GI loss and diuretics (H+ and Cl- interchange). Common causes of HCO3 gain include NaBicarb
ingestion (multiple antacid pills) and milk-alkali syndrome (CaCO3).
* Metabolic alkalosis is a chronic process, with compensatory slowing of respiratory rate and increase in PCO2.
* Metabolic alkalosis has increased pH, increased HCO3, respiratory rate slows to raise PCO2 thereby increasing
pH almost back to a normal value, never overcompensation.
* 67yo woman with HTN is on HCTZ, complains of fatigue. Exam shows orthostatic hypotension. Electrolytes
show low potassium, low sodium, low chloride. ABG is 7.42/47/PO2/33/97%. So PCO2 of 47 is high, and HCO3 of
33 is high. This is metabolic alkalosis defined by high pH and high HCO3. The PCO2 being high is respiratory
compensation.
* Respiratory acidosis caused by hypoventilation (acute or chronic), an inability to blow off CO2, and if sustained
the kidneys will retain HCO3 in an attempt to lessen the acidosis. Acute causes include narcotics, acute COPD, or
asthma. Chronic causes include COPD, obesity, Pickwickian (obesity hypoventilation syndrome), and sleep apnea.
* Respiratory acidosis has decreased pH, increased PCO2, HCO3 unchanged because not enough time. HCO3 rises
with time to compensate almost back to a normal pH.
* 27yo man found unconscious on the street. Respiratory rate is 7. Needle tracks on arms. Pupils small. ABG shows
7.28/56/PO2/28/84%. This is a narcotic overdose with respiratory acidosis due to hypoventilation.
* 72yo woman has COPD worsening over weeks. Comes to the ED because she is becoming gradually SOB. ABG
shows 7.37/5/PO2/34/90%. Her PCO2 is high, so likely a CO2 retainer with the COPD. How do we know this is not
primary metabolic alkalosis (i.e. inc HCO3 with inc PCO2 compensating)? We know this because the pH of 7.37 is
not alkalosis; the body does not overcompensate.
* Metabolic acidosis is the most complicated of the group.
* Metabolic acidosis is a relative increase in the quantity of acid, with causes including addition of acid, inability to
excrete acid, and loss of base (HCO3).
* Metabolic acidosis has decreased pH, decreased HCO3, and increased respiratory rate as compensation in an effort
to blow off PCO2 thereby increased pH almost back to normal, occurs rapidly.
* 17yo with diabetes has not been taking his insulin, has altered mental status. ABG shows 7.29/29/PO2/17/97%.
This is metabolic acidosis as bicarb is low. As compensation, body is breathing fast to bring down PCO2.
* Must determine which type of metabolic acidosis, either high-anion gap (anion gap) or normal anion gap (non-
anion gap). Anion gap is normally 11 +/- 3, so 8-14. Anion gap = (Na + K) – (Cl + HCO3).
* High anion gap acidosis is an excess of non-HCl acid. Normal anion gap acidosis is excess HCl or loss of HCO3.
* Anion gap > 14 caused by overproduction or under-excretion of an acid other than HCl. Chloride levels are
unaffected “normochloremic.” There are four major causes: ketoacidosis (alcoholic, diabetic, starvation), lactic
acidosis, renal failure (uremia), or intoxication (ethylene glycol, methanol wood alcohol, salicylate, others).

DO NOT DISTRIBUTE - 27 -
Study Notes – Internal Medicine James Lamberg 01Feb2010

* MUDPILES: methanol, uremia, diabetic ketoacidosis, paraldehyde/phenformin, iron/INH, lactic acidosis, ethylene
glycol, salicyclates.
* Ketoacidosis is due to overproduction of ketone bodies. Lactic acidosis is due to overproduction of lactic acid.
Renal failure causes inability to excrete organic (non-HCl) acids. Intoxication causes addition of non-HCl acids.
* Anion gap < 14 (normal) usually caused by HCO3 loss in GI tract or urine or inability to excrete acid in urine.
Common causes are diarrhea and renal tubular acidosis (RTA).
* Renal tubular acidosis key clue is the inability to acidify urine, urinary pH typically > 5.4 or even overtly alkaline.
* Type 1 RTA occurs in distal tubules of the kidney. Inability to acidify urine. Seen in kidney stones, amphotericin
B, lithium, sickle cell. Diagnosed by giving acid load/challenge test and still can’t acidify urine. Treatment is oral
bicarbonate.
* Type 2 RTA occurs in the proximal tubules of the kidney. Inability to absorb bicarbonate until levels very low.
Seen in myeloma, Fanconi syndrome, vitamin D deficiency, Wilson disease. Diagnosed by seeing bicarbonate lost in
urine in the face of acidic blood. Treatment is volume restriction.
* Type 4 RTA occurs due to hypoaldosteronism. Occurs with aldosterone or adrenal insufficiency and diabetes.
Presents with hyperkalemia. Diagnosed by high urine sodium with salt restriction (salt restriction test). Treatment is
mineralocorticoid replacement, fludrocortisone.
* 72yo man has 4 days of diarrhea and presents to ED with weakness. ABG shows 7.31/30/PO2/17. Lytes are Na
140, K 2.9, Cl 113, HCO3 18. Is this acidosis or alkalosis? Answer is acidosis. Is this metabolic or respiratory?
Answer is metabolic acidosis, PCO2 being low is compensatory. What is the anion gap? Gap is 12, so normal anion
gap (11 +/- 3). What is the most likely cause of the this patient’s normal anion gap metabolic acidosis? Most likely
diarrhea. Could be RTA. What would be the best test to distinguish diarrhea from RTA? Answer is urine pH, which
should be acidic (< 5.4) in the presence of blood acidemia; urine would be inappropriately high pH in RTA.
Kidney Stones (Nephrolithiasis)
* Urolithiasis occurs in 1-5% of the US population, more common in desert southwest (e.g. Arizona, Nevada).
* Five kinds of stones: calcium oxalate (most common), calcium phosphate, magnesium/ammonium/phosphate
(“triple phosphate” or struvite), uric acid, cysteine (rare).
* Oxalate stones account for about 70%, phosphate about 10%, struvite about 5-10%, urate about 5%, cysteine rare.
* Oxalate, phosphate, and struvite have calcium and are radio-opaque. Urate may not be seen on plain x-ray.
* Main causes of calcium oxalate/phosphate stones include increased calcium absorption (sarcoid, vitamin D
intoxication), idiopathic hypercalciuria, hyperparathyroidism, destructive bone disease (myeloma, metastatic
cancer), hyperoxaluria (fat malabsorption as fat binds to calcium, leaving oxalate for excessive absorption), vitamin
C excess.
* Main causes of struvite stones include infection with urea-splitting bacteria (proteus most common, klebsiella,
pseudomonas), alkaline urine favors crystallization of struvite and can result in large stones (staghorn calculi).
* Main causes of uric acid stones include gout, tissue breakdown (e.g. hematologic malignancies), are radiolucent
and may not be seen by CT or other x-ray.
* Kidney stones present with severe flank pain radiating to the groin, pain onset often sudden, pain may be constant
or spasmodic, nausea and vomiting common. Pain medication may be one of the first priorities.
* 35yo male has sudden onset of severe flank pain radiating to groin, with nausea and vomiting. What is the best
first test to support the diagnosis of kidney stone. Answer is urinalysis for hematuria. The absence of hematuria
argues strongly against kidney stone. What is the next step in determining diagnosis, or what is the best test to
confirm diagnosis? Answer is non-contract helical (spiral) CT scan.
* Spiral CT is the test of choice as it is fast, most sensitive, identifies radio-opaque and even some radiolucent
stones. KUB (kidneys, ureter, bladder x-ray) is useful for patients with known stone disease, difficult to interpret as
gas may be in the way. Sonogram is the test of choice for identifying stones in pregnant women. IVP (intravenous
pyelogram) requires contrast, no longer test of choice, bowel prep needed for good pictures.
* 26yo female presents with sudden onset of severe flank pain radiating to groin, with nausea and vomiting.
Urinalysis shows hematuria. What is the next step in the diagnostic evaluation? Answer is pregnancy test. If
pregnancy test is positive, then do sonogram. If negative, do spiral CT.
* Say the 26yo female is not pregnant and the spiral CT shows a small radiolucent stone in the ureter. What is the
most appropriate initial treatment? Answer is analgesics (e.g. morphine) and high flow IVF.
* For small stones (<5mm) in lower ureter is observation. Small stones at the ureteropelvic junction (UPJ) almost
always pass. Large or with significant obstruction requires surgical intervention.
* Ureteroscopy involves putting cystoscope through urethra up into bladder then fragments stones and pulls them
out with forceps or baskets.
* Lithotripsy (ESWL) can be used for small renal (<2.5cm) and ureteral (<1.5cm) stones.

DO NOT DISTRIBUTE - 28 -
Study Notes – Internal Medicine James Lamberg 01Feb2010

* Percutaneous nephrostomy is used for obstructed stones to drain the kidney via a tube and prevent backflow. It can
be combined with lithotripsy.
* For complex stones (e.g. struvite) or extensive anatomic abnormalities, open surgical removal is performed.
* You should always try to determine the type of stone the patient has. To do this you strain the urine and send for
urinalysis. So the patient will urinate at home into a funnel until a stone or particles are found.
* Initial tests in a “stone workup” include looking for increased calcium absorption (24h urine calcium), idiopathic
hypercalciuria (24h urine calcium), hyperparathyroidism (serum Ca level, PTH level), destructive bone disease
(serum Ca level), fat malabsorption (history, greasy stools), vitamin C excess (history).
* Treatment for increased calcium absorption (thiazides), idiopathic hypercalciuria (thiazides), hyperparathyroidism
(treat problem), destructive bone disease (treat problem), fat malabsorption (treat problem), vitamin C excess
(discontinue vitamin C).
Renal Cysts
* Three types of cystic kidney disorders: polycystic kidney disease (PKD) in autosomal dominant form (1:300 to
1:1000) and autosomal recessive form (rare, infancy), simple cysts (incidental), complex cysts (rule out cancer).
* PKD is autosomal dominant in 90% of cases, 10% are sporadic mutations without family history.
* PKD causes 5-10% of end stage renal disease needing dialysis. Cause is uncertain.
* PKD involves multiple renal cysts, asymptomatic in childhood, symptoms develop in 20s and 30s including pain
from kidney size pushing on renal capsule, hematuria, infections, stones, high blood pressure via RAAS.
* PKD manifests with hypertension in the kidneys, impaired circulation due to distortion of renal vasculature leads
to increased renin-angiotensin. Renal failure occurs with progressive decline in function, half have ESRD by age 60.
Stones and infection also occur.
* Extra-renal manifestation of PKD include hepatic cysts (50%), also pancreas, ovaries, spleen cysts. Intracranial
aneurysms (10%) berry aneurysms that can lead to subarachnoid hemorrhage. Mitral valve prolapse (25%). Colon
diverticula more often than in general population.
* PKD diagnosed with renal ultrasound, diagnose symptomatic patients, screening of relatives at risk, only 2/3 have
cysts in their teens, cysts not universally present until about 30s.
* Genetic diagnosis can be done with prenatal amniocentesis and chorionic villus sampling (CVS). For adults and
children at risk, gene linkage studies are useful if considering kidney donation to affected relative.
* Treatment of PKD is manage complications, control hypertension, treat UTIs/stones, dialysis/transplant for ESRD.
* 32yo man is seen for hypertension and hematuria. Tells you that father and cousins had a similar problem and both
died of hypertension-related renal failure. Creatinine is 1.5. What is the best test to make a diagnosis? Answer is
renal ultrasound to rule out PKD.
* Simple renal cysts are usually incidental findings on sonogram or CT. Can be single or multiple, usually single. Is
a homogenous cysts without echos (fluid filled sac). No treatment is needed unless large or symptomatic.
* Complex renal cysts are usually incidental findings on sonogram or CT, or found during workup of hematuria or
abdominal pain. By definition, a complex cyst is cancer until proven otherwise. Must do a percutaneous aspiration
biopsy (CT needle aspiration), regardless of size.
* Complex renal cysts vary in size, multiple (complex) echos/septations seen on CT or ultrasound.
* Again, complex renal cysts are cancer until proven otherwise and need CT-guided needle biopsy.
* 62yo man is being evaluated for RUQ abdominal pain. Sonogram to study gallbladder reveals gallstones, but also
a small (2cm) cyst on left kidney with complex echos. CT scan confirms findings. Patient has no symptoms from left
kidney, and no hematuria. What is the next step in the evaluation. Answer is cyst aspiration/puncture.
Hypertension (HTN) & Hypertensive Emergencies
* Optimal is systolic < 120 and diastolic < 80. Normal is systolic < 130 and diastolic < 85.
* Hi-normal is systolic 130-139 or diastolic 85-89.
* Stage 1 hypertension is systolic 140-159 or diastolic 90-99.
* Stage 2 hypertension is systolic 160-179 or diastolic 100-109.
* Stage 3 hypertension is systolic 180+ or diastolic 110+.
* Epidemiology shows 50million Americans with HTN, that is 1/4 of the adult population. 60% of persons over age
65 have HTN and 75% are not controlled.
* High risk groups for HTN include African Americans, obese, diabetics, and males > females.
* Most common presentation is incidental finding in an asymptomatic individual. Hypertensive emergency is
uncommon. Chronic complications can develop; patient may present with heart attack or retinal hemorrhage.
* Chronic complications include heart disease with >1million MIs per year, 400,000 new CHF per year.
* Chronic complications include CVA (400,000 strokes/year), renal failure (5,000 ESRD/year), eye disease.

DO NOT DISTRIBUTE - 29 -
Study Notes – Internal Medicine James Lamberg 01Feb2010

* Essential hypertension has no clearly identified cause or reversible process. < 5% of HTN is secondary, due to
some other condition that is often reversible.
* BP measurement method: patient resting quietly for 5 minutes, seated with arm at heart level, bladder encircles
80% or more of arm, average of 2 readings, confirm on 2 subsequent visits.
* “White-coat” hypertension is thought to occur in about 20% of cases of diagnosed HTN.
* Ambulatory BP monitoring gives data for a 24h period, useful for suspected “white coat” hypertension.
* After you get a first blood pressure reading, when do you have to get the patient back for a confirmation? If BP
systolic > 180 or diastolic > 110, then 1 week. If BP systolic 160-179 or diastolic 100-109, then 1 month. If BP
systolic 140-159 or diastolic 90-99, then 2 months. If BP systolic 130-139 or diastolic 85-89, then yearly. If BP
systolic < 130 or diastolic < 85, then every other year.
* Hypertensive emergencies need immediate treatment: angina or CHF, cerebral impairment (ischemia,
subarachnoid hemorrhage, severe headache), acute eye findings (hemorrhages, papilledema, blurred vision).
* Treatment for hypertensive emergency is nitroprusside infusion. Also nitroglycerine (with MI), labetalol, enalapril.
* 32yo previously normotensive man sees you for a work physical, BP is 170/99. His history and exam are
otherwise unremarkable. What is the next step in evaluating his blood pressure? Answer is return in 1-2 months.
* Say the patient comes back and their HTN is confirmed. Perform a physical exam and laboratory investigations to
seek secondary causes, seek end-organ damage, and seek/control atherosclerotic (ASHD) risks.
* Investigating secondary causes can be done by history, exam, and basic labs. Medication causes include birth
control pills and alpha-agonists (e.g. seasonal rhinitis spray). Hyperthyroidism can cause HTN (tremor, goiter).
Coarctation of the aorta can be found by pulse lag, pulse striking earlier in radial than femoral arteries. Renal artery
stenosis is found by bruit on abdominal exam or sudden abrupt onset of HTN. Polycystic kidneys can be found by
mass or hematuria. Cushing disease will have striae, buffalo hump, rounded moon-facies. Hyperaldosteronism will
come with low potassium. Pheochromocytoma presents with sympathetic episodes. Renal failure can be discovered
by creatinine level elevation. Aortic regurgitation has high systolic pressure and low diastolic pressure, murmur.
* So for secondary HTN causes, do history, physical, creatinine, potassium.
* What is the best test if a secondary cause is suspected? Medication-induced is stop meds and follow BP,
hyperthyroid is ultra-sensitive TSH, coarctation is ultrasound studies, renal artery stenosis is captopril renogram in
young (fibromuscular) and Doppler flow in elderly (atherosclerotic), polycystic kidney is renal sonogram, Cushing
is dexamethasone suppression test and/or cortisol, aldosteronism is aldosterone level, pheochromocytoma is VMA
and metanephrines and catecholamines, renal failure is BUN/Cr, aortic regurgitation is echocardiogram.
* What is the initial treatment if a secondary cause is suspected? Medication-induced is stop meds and follow BP,
hyperthyroid is beta-blockers and anti-thyroids, coarctation is surgical, renal artery stenosis is angioplasty,
polycystic kidney is ACE-I (maybe) and renal transplant, Cushing is remove pituitary adenoma, aldosteronism is
remove aldosterone tumor, pheochromocytoma is alpha-blockade and surgery, renal failure is renal transplant, aortic
regurgitation is ACE-I and valve replacement.
* 45yo previously normotensive man sees you for work physical and BP is 170/99. Multiple BP readings over 2-3
months confirm hypertension. History is unremarkable except vague LUQ abdominal pain for past 6 months, exam
shows LUQ fullness. Urinalysis shows hematuria. Other labs are normal including creatinine. What is the most
appropriate test for a secondary cause of hypertension? Answer is renal ultrasound to rule out polycystic kidney.
* End-organ damage can be investigated by looking for retinopathy, carotid bruits, CHF, ECG (vs echo vs nothing)
for left ventricular hypertrophy, AAA (exam vs sono), lower extremity pulses.
* Risk factors for atherosclerotic heart disease (ASHD) are smoking, sedentary lifestyle, obesity, diabetes,
hyperlipidemia, newer risk factors (C-reactive protein, homocysteine levels).
* Homocysteine levels can be lowered with vitamin B6, vitamin B12, folic acid, no evidence it improves heart
disease. C-reactive protein level also not proven to be linked to heart disease, but being investigated.
* 45yo previously normotensive man sees you for work physical and BP 165/92. Multiple BP readings over 2-3
months confirm hypertension. History reveals 1PPD smoker for 30 years, otherwise no ASHD risk factors. Physical
exam and all lab tests are unremarkable. What is the best initial step in his treatment? Answer is smoking cessation.
* Hypertension treatment initially for stage 1 and maybe stage 2 is lifestyle modification: weight reduction, exercise,
smoking cessation, salt reduction, possibly alcohol reduction is in large amounts.
* Medications are first-line for uncomplicated essential hypertension (stage 3, most stage 2, stage 1 if it does not
respond to lifestyle modification). Diuretics (thiazides) are first or beta-blockers.
* Special circumstances: ACE-I used in diabetes, mild renal insufficiency, CHF due to systolic dysfunction, or
recent MI. ACE-I will improve renal blood flow a bit and lower risk of progressive renal damage.
* Calcium-channel blocker used in isolated systolic hypertension in elderly, possibly better than beta-blockers in
African American population if adding to diuretic.

DO NOT DISTRIBUTE - 30 -
Study Notes – Internal Medicine James Lamberg 01Feb2010

* “Double-duty situations”: calcium-channel blocker for Raynaud, SVT, atrial fibrillation. Alpha blocker for
prostatic hypertrophy. Beta-blocker for migraine headache.
* Hypertension in pregnancy, recommended drugs: methyldopa is first line. Labetalol or hydralazine also
acceptable. Contraindicated drugs include ACE-I and angiotensin receptor blockers (ARBs), teratogenic.
* What if you start treating a patient and the hypertension is not responding. Most common cause is non-
compliance, patient may be confused about treatment regimen. Consider alcohol abuse and NSAIDs. Re-consider
secondary causes.
* Osler phenomenon, usually in elderly, is artery calcification that prevents full compression of the artery by the
blood pressure cuff, thus giving a falsely high reading. Clue to this palpable artery with inflated cuff.

Kaplan Videos – Infectious Disease with Dr. Conrad Fischer, MD


The organisms that cause certain diseases do not change over the years. The antibiotics do however.
Overview of Antibiotic Therapy
* Most common cause of pyelonephritis is E. coli, most common cause of osteomyelitis is S. aureus.
* So what antibiotics go along with staph/strep, gram negative rods, and anaerobes?
* If you do PCR resting on Egyptian mummies from 5 thousand years ago, you will find there was beta-lactamase in
the staphylococcus.
* Ampicillin, penicillin, amoxicillin alone is good for streptococcus, but not for staphylococcus.
* Antibiotics for staphylococcus and streptococcus are oxacillin, cloxacillin, dicloxacillin, nafcillin (not methicillin).
* With methicillin-sensitive staphylococcus, what do you use? Not methicillin. Use ox, clox, diclox, naf. Why don’t
we use methicillin? Because it causes interstitial nephritis.
* Another alternative for staphylococcus and streptococcus (e.g. penicillin allergy) is a beta-lactam drug, first-
generation cephalosporins like cephalexin, cephadroxil, cefazolin. 5% cross-reaction with penicillins at most and
almost never anaphylaxis, so don’t worry if the allergy is a little rash.
* What if patient says they have a penicillin allergy, that causes a little rash, and hypotension, and stridor, and
laryngeal edema so they had to get intubated, then got IV steroids, then dopamine drip in the ICU. For life-
threatening penicillin allergies, then we should avoid all the beta-lactam antibiotics.
* Macrolides: erythromycin, azithromycin, clarithromycin. Macrolides with clindamycin will cover staph/strep, but
are not first choice, choice for life-threatening penicillin allergy. Vancomycin works, but should be reserved for
methicillin-resistant staph aureus (MRSA). Vancomycin not superior in efficacy to ox/clox/diclox/naf, just superior
in its resistance pattern.
* Gram negative rods include E. coli, klebsiella, proteus, enterobacter, citrobacter, pseudomonas.
* Antibiotics for Gram-negative rods are aztreonam, aminoglycosides (gentamycin, streptomycin, tobramycin,
neomycin, amikacin), fluoroquinolones (ciprofloxacin, levofloxacin, trovafloxacin, gatifloxacin, sparfloxacin,
moxifloxacin), second-generation cephalosporins (will not cover pseudomonas), third-generation cephalosporins
(ceftazidime, ceftriaxone), carbapenems (imipenem, meropenem), extended spectrum penicillins (piperacillin,
ticarcillin, azlocillin, mezlocillin).
* Carbapenems cover Gram-negative and Gram-positive, but you should not use them for Gram-positive infections
(like strep throat) because there are better drugs exclusively for Gram-positives. “Do not swat a fly on your friend’s
head with a hammer.”
* TMP-SMX is only good for uncomplicated cystitis and prophylaxis against pneumocystis pneumonia.
* Chloramphenicol is never the right answer, it causes aplastic anemia and Gray Baby Syndrome.
* Cephalosporins range from strong Gram-positive coverage at first-generation (poor Gram-negative) to strong
Gram-negative coverage at third-generation (poor Gram-positive). Fourth-generation cephalosporins (cefepime) are
a combination of first-generation and third-generation, so they cover Gram-positives and Gram-negatives.
* Antibiotics for anaerobes are metronidazole (particularly good for abdominal), clindamycin (does not cover bowel
well), carbapenems as well. If you have an exclusively anaerobic infection, don’t jump to imipenem.
* Tetracycline is not the correct choice anymore; use doxycycline instead. Doxycycline used for Lyme (mild
disease: rash, facial nerve palsy), Chlamydia, rickettsia, as these are intracellular organisms.
* Nitrofurantoin used for UTI in pregnancy, no other use.
Meningitis
* 57yo man comes to the ED with fever and a headache. Photophobia or seizures or nausea or vomiting are too non-
specific here. A specific additional symptom to lead you down the right path would be stiff neck (nuchal rigidity),
implying meningitis. What if they want you to know it’s a brain abscess? Fever, headache, and focal findings. What
about encephalitis? Fever, headache, altered mental status (confusion, encephalopathy).

DO NOT DISTRIBUTE - 31 -
Study Notes – Internal Medicine James Lamberg 01Feb2010

* So 57yo man has fever, headache, stiff neck. What is the first test? Do you do a lumbar puncture or a CT scan?
What is the first test? Lumbar puncture (LP). Only do a CT scan prior to a spinal tap if papilledema or focal findings
that would imply a brain mass/abscess. Patient also needs to be able to follow commands, such as “squeeze my
hands,” because you cannot determine focality. If you had a mass lesion that was so small it did not cause focality,
an LP would not cause herniation.
* If you are going to do a CT prior to the LP, you should start the patient on antibiotics first (dose of ceftriaxone).
Doesn’t that ruin the sensitivity of the LP culture? Yes, but you’d rather be alive with a clean LP than dead with an
accurate LP. Other information from the LP can help tell you the diagnosis.
* The most common neurologic problem after meningitis is deftness, CN VIII damage, it can happen within hours.
* With papilledema you worry about herniation. “You go to do a rectal later and say ‘my that’s a smooth prostate’
and the patient says, ‘that’s not my prostate, that’s my pons.’”
* When you have infectious diseases, what is the most accurate diagnostic test? Answer is culture. All other tests are
compared to culture. When do you wait for results of the culture? Never.
* CSF analysis: Which form of meningitis can give an elevated protein and decreased glucose in the LP? Answer is
all. Which gives it most often? Bacterial, strep pneumonia. Gram-stain is 50-60% sensitive, so won’t be positive
most of the time. So what will tell you right now what type of organism you have? Answer is cell count.
* CSF cell count: Only bacterial gives you 1000s of polys, where others give 10-100s of lymphocytes. If you see
1000s of polys, give ceftriaxone. What about 10-100s of lymphocytes? Could be fungal (cryptococcus), Rocky
Mountain Spotted Fever, Lyme disease, syphilis, tuberculosis, viral meningitis.
* For cryptococcus meningitis, expect HIV patient with < 50 CD4 T-cell count. Best initial test? Answer is not India
Ink. However, India Ink is about 50% sensitivity (good if positive, means nothing if negative). Answer is
cryptococcal antigen testing, positive in 95-98%, very sensitive and specific. Should you start this patient with
fluconazole? No, use amphotericin B. Amphotericin B (better than fluconazole for saving life in meningitis), then
follow up with fluconazole forever. Fluconazole is continued life-long, else cryptococcal meningitis recurs.
* Rocky Mountain Spotted Fever (RMSF) is not seen on Gram stain because it is intracellular (rickettsia). Lyme also
not seen on Gram stain because it is a spirochete. You also can’t see TB, viral, Legionella, or syphilis on Gram stain.
* So, CSF from lumbar puncture shows 10-100s of lymphocytes, Gram stain negative, culture negative. Now what?
Answer is serologic testing to look for specific antibody against Lyme and RMSF. This helps you determine if you
should use doxycycline or ceftriaxone. So this is very easy, right? We just get some IgM acutely and IgG
chronically. Remember the Weil-Felix test? Forget it.
* So are with “atypical” results (Gram stain negative, culture negative), are we just going to order Lyme, RMSF,
cryptococcal antigen testing, AFB (acid-fast bacillus) stain and culture, viral serologic testing, syphilis serology with
a VDRL and FTA on all of them? Nope. You have to suspect the specific disease.
* If RMSF, centripetal (moves centrally) rash and outdoor activity. If Lyme, target rash (erythema migrans) and
outdoor activity. Lyme in Connecticut, Massachusetts, New York, New Jersey (North East). RMSF in Alabama,
Kentucky, Tennessee, Carolina, it shouldn’t matter, look for the centripetal rash. If TB, diagnose with TB stain and
AFB cultures. If viral, diagnosis is by exclusion of the others, there is no treatment.
* Test TB (meningitis and pulmonary) with TB stain, TB culture, INH, rifampin, pyrazinamide, ethambutol.
* When do you answer steroids for meningitis in adults? Answer is TB meningitis. Some mild benefit possibly with
steroids given prior to starting antibiotics and LP.
* When do you answer intrathecal antibiotics? Answer is never, not necessary. Methotrexate is given intrathecal in
ALL (acute lymphoblastic leukemia). Intrathecal antibiotics is like intra-cardiac epinephrine, nifty idea but not
necessary.
* Elderly neonate is HIV positive, on steroids for CLL, has a lumbar puncture with elevated protein and decreased
glucose, Gram stain negative, 3200 polys. What is the next best step in the management of this patient? Do we give
amphotericin or ceftriaxone? HIV is a risk factor for fungal meningitis. This isn’t fungal because there are polys
present. This patient is immunocompromised. So what organism do we have to cover? Answer is listeria. Answer
for management is give ceftriaxone and ampicillin, because listeria is resistant to all forms of cephalosporins.
* Things that put you are risk for listeria include immunocompromised, neutrophil and T-cell defects, HIV (T-cell
defect), steroids (neutrophil and T-cell defect), leukemia/lymphoma elderly or neonate then do empiric coverage.
Encephalitis
* Fever, headache, and confusion (encephalopathy). What is the next step? Head CT. The head CT will most likely
be normal. Lots of encephalitis in the world, eastern equine encephalitis, western equine encephalitis, Colorado tick
fever, Congo Crimean fever, Venezuelan hemorrhagic fever, Bolivian hemorrhagic fever, viral encephalitis.
* If you add all those cause of encephalitis and multiply by ten, they still are not as common as herpes encephalitis.
If the case is clearly encephalitis and they ask diagnosis, your best bet is to answer herpes.

DO NOT DISTRIBUTE - 32 -
Study Notes – Internal Medicine James Lamberg 01Feb2010

* Lumbar puncture should come with PCR, which has largely replaced the biopsy. What is suggestive of herpes on a
lumbar puncture? Answer is increased red cells, mildly hemorrhagic. How do you know it is not a subarachnoid
hemorrhage? There will also be increased WBCs from infection, plus clear CT scan favors against subarachnoid.
* Treatment of herpes encephalitis is acyclovir.
Brain Abscess
* Fever, headache, and focal findings. Next step is CT scan (or MRI). Head CT will show ring- or contrast-
enhancing lesions. Two groups of diseases that give ring-enhancing lesions are cancers and infections. Staph, strep,
glioblastoma, anaerobes, toxoplasmosis, oligodendroglioma, astrocytoma.
* Next step depends on HIV status. If patient is HIV positive, assume toxoplasmosis and give pyrimethamine and
sulfadiazine for a couple of weeks and repeat CT scan. In HIV, 90% of the time it is toxoplasmosis or lymphoma. If
the CT scan is the same, do a biopsy to see what is at the ring-enhancing site.
* If patient is HIV negative, much broader range of disease. Next step is not lumbar puncture because the patient
will herniate and die. Next step is brain biopsy; the only way to know what it is.
* So again, we have a lesion with mass effect giving focal findings. Do not do LP. Do a CT to see ring-enhancing
lesion. Then do a biopsy. You don’t want to start antibiotics and have the patient come back with a permanent
deficit saying, “doc, I heard antibiotics don’t work well for glioblastoma.” Or you radiate the patient for 6 weeks and
they say, “doc, the radiation isn’t working for my staph aureus.” You have to biopsy to know.
Otitis Media & Sinusitis
* What is the most sensitive physical finding for otitis media? Answer is immobile tympanic membrane on
insufflation. You may see a red bulging tympanic membrane, occasionally fever, pain on pulling pinna.
* Treatment is amoxicillin, based on American Society of Otolaryngology, half the time the otitis media resolves on
its own because they are viruses, if you haven’t recently been treated (i.e. resistance) the efficacy of amoxicillin is
just as good as any of the other antibiotics.
* Second and third generation cephalosporins, cefuroxime, cefixime, cefaclor would work. Macrolides
azithromycin, clarithromycin work. Amoxicillin/clavulanic acid works too. If not child, new fluoroquinolones like
levofloxacin, moxifloxacin, gatifloxacin work as they cover strep pneumonia.
* 28yo female with headache, rhinorrhea, sinus tenderness, tooth pain, decreased transillumination of sinuses. What
is the next step in management. Answer is give amoxicillin.
* Acute otitis media (AOM) is about 40% strep pneumonia, 30% h. influenza, 20% moraxella catarrhalis.
* What is the most accurate diagnostic test for this sinusitis patient? Answer is culture.
* The best initial diagnostic test is an x-ray. A CT is more accurate than an x-ray, but not first.
Pharyngitis
* Symptoms are sore throat, adenopathy, and exudate. Answer for treatment is oral penicillin, Pen VK. Most
accurate test is Gram stain. Why do you not do a Gram stain for pharyngitis? It will not tell you if you have strep
pyogenes; it can’t tell you what type of strep is there.
* The rapid strep tests are latex agglutination tests, looking for the surface antigens of strep pyogenes.
* Why don’t we Gram stain the vagina? Gardnerella is normal flora, so we need clue cells to see when there is too
much. We Gram stain sputum because strep pneumonia with lancet-shaped diplococci are not normal flora in the
mouth. We do Gram stain the cervix to look for neisseria gonorrhea.
* If patient is allergic to penicillin, what do we use for pharyngitis? Second or third generation cephalosporins,
azithromycin, clarithromycin, levofloxacin, moxifloxacin, gatifloxacin.
* Erythromycin has an advantage over the others as it can prevent rheumatic fever.
Bronchitis & Pneumonia
* Fever, cough, with or without sputum. You can’t say this is influenza, pneumonia, bronchitis. All you know is this
is a respiratory infection. What test should you order first? Answer is x-ray, first for respiratory infections. If CXR is
normal (no infiltrates, effusions, consolidations), then this is bronchitis.
* Treatment for bronchitis (with sputum) is second or third cephalosporins, azithromycin, clarithromycin,
levofloxacin, moxifloxacin, gatifloxacin. Ampicillin would work also.
* What is the best diagnostic test? Answer is biopsy/aspiration.
* How do we know if the patient has an abscess? Halitosis due to anaerobes. To get an abscess, the patient must
have an increased risk for aspiration and an oral infection.
* Intubation aspiration, stroke, poor dentition, intoxicated, loss of gag reflex, you increased your risk of aspiration.
* For anaerobes above the diaphragm we use clindamycin. For below the diaphragm we use metronidazole.
* Why don’t we Gram stain or culture sputum for an abscess? Everyone has anaerobes in their sputum.
* How do we determine the types of pneumonia if infiltrates are found on CXR? There will be lobar infiltrates or
bilateral interstitial infiltrates.

DO NOT DISTRIBUTE - 33 -
Study Notes – Internal Medicine James Lamberg 01Feb2010

* Lobar infiltrates is regular bacterial pneumonia, strep, hemophilus, staph, klebsiella. If bilateral interstitial
infiltrates, could be mycoplasma, chlamydia, legionella, viral pneumonia, pneumocystis pneumonia.
* Young healthy person admitted for lobar pneumonia is pneumococcus. Elderly patient with lobar pneumonia is
pneumococcus. Elderly patient with lobar pneumonia and has COPD or smokes often, then it is strep pneumonia.
* Pseudomonas pneumonia seen in the ICU in intubated patients, hospital acquired infection.
* When do you wait for the results of culture? Answer is never. Get culture and start antibiotics.
* Why is sputum stain not first? Even when you do the culture, you don’t get the diagnosis half the time.
* In long-term alcoholics with lobar pneumonia, most common is strep pneumonia. In smokers and COPD patients
with lobar pneumonia, most common is strep pneumonia. Klebsiella pneumonia is more likely to be seen in
alcoholics (but not most common). H. influenza most likely to be seen in smokers and COPD patients (but not most
common).
* Mycoplasma pneumonia usually seen in young, healthy, patient, “walking pneumonia.” Mycoplasma not seen on
Gram stain (no cell wall) and not grown in regular culture. Atypical means it is not seen on a Gram stain and not
grown in regular culture. Look at IgG and IgM serology. Cold agglutinin testing for mycoplasma is not sensitive or
specific enough, so not a good way to make a diagnosis.
* Treatment for mycoplasma or legionella is macrolide (Erythromycin, Azithromycin, Clarithromycin) or quinolone.
Doxycycline also works.
* Patient has bilateral interstitial infiltrates. Test for legionella pneumonia (diagnosis) is urine antigen. Urinary
antigen for legionella is 99% sensitive and specific. If you don’t screen for it with the test, you will not see the
diagnosis. It accounts for about 5-10% of community acquired pneumonia. Many people will not think about
ordering this type of test unless they know it exists. A patient presents with lots of coughing and you’re getting a
urine test; it doesn’t seem logical unless you understand the test.
* How will you know who to do a legionella urinary antigen test for? Not a person exposed to air conditioning or a
person exposed to water. Look for a patient with GI and CNS problems; confusion, diarrhea, abdominal pain.
* Most specific test for legionella is buffered charcoal yeast extract agar (BCYE), but it takes a couple of weeks.
Antibody testing also takes weeks and is never available to make a treatment decision.
* What is the fastest way to increase the incidence of sexually transmitted diseases in your community? Answer is
bring in a STD specialist into your community, they’ll test everyone, and the incidence will rise. People will be like,
“Hmm, ever since this new guy came to town everyone is getting Gonorrhea...”
* Any cause of pneumonia can cause hyponatremia. Anything in the lung can cause SIADH. Anything in the brain
can also cause SIADH.
* With pneumocystis pneumonia, patient is HIV positive with < 200 T-cells. Presenting symptom is usually
dyspnea. CXR shows bilateral interstitial infiltrates. PO2 is 65. What is the next best step in the management of this
patient? Answer is oxygen, then give TMP-SMX (co-trimoxazole). Steroids will increase the inflammation and thus
increase the ability to transfer oxygen across the interstitial membrane. But isn’t it dangerous to give steroids to
someone who is immunocompromised? Maybe, but isn’t it better than dying of hypoxemia today? You’d rather be
alive with re-activated TB then dead with no TB. Also, short-course steroids for 2-3 weeks will not hurt.
* Steroids used if hypoxemia is severe. Severe is PO2 < 70.
* On hospital day 2, this HIV patient develops the most common side-effect of IV TMP-SMX. That side-effect is a
diffuse maculopapular hypersensitivity rash. How what do you put the patient on? Answer is pentamidine. What is
the major side-effects of IV pentamidine? Answer is pancreas problems. Pentamidine is only second-line if patient
has IV therapy. For prophylaxis, the second-line is dapsone. But there is no IV dapsone.
* Patient has bilateral infiltrates after sniffing a placenta, has cough. Sniffing a sheep (Yemenite sheep) placenta.
Serology diagnoses Q-fever (coxiella burnetii). Treatment is doxycycline. Coxiella is the only airborne rickettsia.
You would need the animal exposure history. So people around animals at the time or birth, the coxiella aerosolizes
at the time of birth and they get the pneumonia. Without that history, you can’t figure it out.
Tuberculosis (TB)
* 37yo male prisoner at maximum security prison. There are over 2 million in prison in the U.S. Up to 60% of
prisoners are PPD positive (tuberculosis), due to ventilation and crowds that you get there. Patient has fever, cough,
sputum, weight loss, night sweats. What is the next best step in the management of this patient. Answer is CXR. It
shows an apical cavity and infiltrate. You put the patient in isolation. Now, what is the next best step? Answer is
sputum, not PPD skin test. You check sputum because PPD is not to diagnose acutely ill patients. A interferon-
gamma quantification test (QuantiFERON-TB) may be used. PPD is used for screening the asymptomatic, for
screening populations at risk.
* Say sputum is positive for AFB (acid-fast bacteria) stain. What is the next best step? Answer is treatment. How
many drugs should patient’s be started on? 4: isoniazid (INH), rifampin (RIF), pyrazinamide (PZA), ethambutol.

DO NOT DISTRIBUTE - 34 -
Study Notes – Internal Medicine James Lamberg 01Feb2010

You give 4 drugs because of the possibility of resistance.


* Which TB drug cause hepatotoxicity? Answer is all of them.
* Which TB drug causes hyperuricemia? Pyrazinamide. What do you do for this patient? Answer is nothing in
asymptomatic hyperuricemia, at any time.
* Ethambutol affects what organ? Eye, optic neuritis. Ethambutol, Eye.
* Abnormal red color with all your body fluids with rifampin. Red, Rifampin.
* Neuropathy occurs with isoniazid. Neuropathy, iNh. INH causes increased urinary loss of vitamin B6.
* Tuberculosis is essentially an economic disease. The amount of TB in a country is proportional to the amount
spent on public health.
* If your PPD is positive, what does this mean? It means you have been exposed. How much risk do you have of
developing the disease? Answer is 10% in a lifetime. Except in HIV, where it is 10% per year. That means 90% of
people who are PPD positive, never develop TB. That means 10 people will get isoniazid for 9 months to prevent
that one single case.
* What is a positive PPD? 10mm of induration. What about 10mm of erythema? No. What about 20mm of
erythema? No. What about 2 meters of erythema? Nope. Induration is the only important point. Except 5mm
induration in HIV positive patients because immune system cannot make it to 10mm.
* What effect does BCG (Bacillus Calmette-Guérin vaccine) have on these recommendations? None. What? If I’m
from India I don’t want to eat isoniazid for 9 months, that’s for sick people, not me. Sorry, it doesn’t matter no
matter how much you think it does. BCG will not give you 10mm of reactivity.
* PPD is 5mm induration in HIV, close contacts, steroid use, organ transplant recipients.
* 71yo woman who has never been tested, PPD is 16mm. CXR is negative. No symptoms. What is the next step in
management. Answer is 9 months of INH. What about a 3,791yo Egyptian mummy, found to be PPD positive,
asymptomatic, negative CXR? What do you do? 9 months of INH. What if the mummy had BCG? 9 months of INH.
* Interferon gamma testing is not affected by BCG at all.
* Who gets the 15mm cutoff for PPD testing? People who should have never been tested in the first place. Not HIV,
not homeless, not healthcare workers, not IV drug users, not any form of disease, just normal people. People who
living in Wyoming (100% sensitivity to TB medications, few cases, always sensitive).
* If PPD is positive, you do a CXR. If CXR is negative, you get INH for 9 months. If positive, you check the
sputum to make sure you’re not dealing with active TB. If sputum is negative, you give INH for 9 months. If sputum
shows TB, you give the 4 drugs, INH, RIF, PZA, ethambutol (EMB).
* If the patient is asymptomatic and you get a positive PPD, you just caught early reactivation of TB.
Infectious Diarrhea
* 27yo female medical student has a lunch today at 3pm, Chinese rice. At 6pm, she has a big, bloody, smelly,
watery, poopy, diarrhea, with a temp of 102 and abdominal pain. What is the cause of her diarrhea? Not staph
aureus, not bacillus cereus, not E. coli, not salmonella, not shigella, not viral, not protozoal (crypto, giardia). Most
likely cause is campylobacter.
* When you compare risk factors with individual presentation, the most important is individual presentation. This
patient has blood in their stool. It could not be viral, protozoa, staph, B. cereus. It could be campylobacter,
salmonella, shigella, E. coli, Yersinia. Risk factors do not matter if the patient has blood per rectum and a fever. C.
difficile can give you blood, but patient has no antibiotics in the history, most of the time it does not cause blood.
* Also, the last thing you ate is not necessarily the thing that gave you diarrhea. This patient got her diarrhea from
what she had from dinner the night before or what she had for lunch the day before. Yes, staph and B. cereus cause
symptoms in 2-6 hours, but they predominately present with upper GI symptoms, like vomiting. Viral is often seen
in children.
* If you are given no history and bloody diarrhea, campylobacter is statistically the most likely. E. coli will be
associated with thrombocytopenia, renal failure, HUS. Yes the travelers and undercooked red meat. HUS is a rare
and uncommon association, but you need to know it. For salmonella, association is poultry. Shellfish association is
Vibrio parahaemolyticus. Campylobacter is the most common cause of invasive diarrhea in the U.S.
* What if you do not get blood in the history? Check leukocytes, WBCs. The only way to distinguish the cause of
bloody diarrhea is by culture.
* C. difficile associated with antibiotics. Test is via toxin. Why not culture? It is difficult to culture. That is why it is
named difficile (Spanish for difficult). Treatment is metronidazole first, not vancomycin.
* Protozoans, you look for ova and parasites in the stool. HIV associated diarrhea is cryptosporidium and there is no
therapy and there isn’t going to be a therapy.
* Giardia associated with camping and homosexual men. Treatment is metronidazole.

DO NOT DISTRIBUTE - 35 -
Study Notes – Internal Medicine James Lamberg 01Feb2010

* Which form of food poisoning (diarrhea) causes symptoms the earliest after ingestion? Answer is scombroid. This
also is associated with wheezing and a rash. It is from spoiled/rotten fish, tuna, mahi-mahi, sardines. The fish eats
organisms that make histamine in the flesh. Treatment is anti-histamines.
* Summary: individual presentation of the patient matters the most. Yes, for basic science we learned staph aureus
and bacillus cereus is 2-6 hours, clostridium perfringens is 12 hours, salmonella/shigella and campylobacter is 24-36
hours. Yea, but that is after you’ve already diagnosed it and you did an outbreak investigation, not in an individual
patient. Legionella and air conditioners, yea after you’ve diagnosed it and we culture all the air-conditioning
systems, but not when the patient is walking in. Your decision point with diarrhea and food-poisoning is blood or no
blood, then white cells or no white cells.
* When you have to treat the non-blood diarrhea causes, use ciprofloxacin. You treat if they have very severe
disease, like hypotension, febrile, septic. Isn’t erythromycin better for campylobacter? Yes, but you don’t know if
it’s campylobacter, because you have to wait for the culture results.
Hepatitis
* Hepatitis A, B, C, D, E, G. There was a hepatitis F for a little while, but they found it was C. Hepatitis G “is a
virus with an identity crisis.” It doesn’t make you turn yellow, get transaminitis, lose weight, make dark urine, or
make light stool. Hepatitis G does not cause clinical disease. A patient is found to test positive for hepatitis G after a
blood donation, what should you tell the patient to expect? Answer is nothing.
* Hepatitis A transferred via food and water (fecal-oral). Can you get hepatitis A sexually? Well, it depends on what
you eat. Hepatitis B, C, and D through blood, sex, perinatal. Hepatitis E (enteric) through food and water. HepE in
the East (SE Asia, Cambodia, Vietnam, Laos), through food and water. HepA is most common in the U.S.
* With acute hepatitis, you get increased ALT, increased bilirubin, malaise, fatigue, tiredness, dark urine (bilirubin
in urine), light stool (bilirubin not in stool anymore), hepatomegaly, splenomegaly, weight loss (turns off appetite).
* Patient presents and is yellow. How do you distinguish via history and physical exam which type of hepatitis we
have? You can’t. What are you going to ask, how long was your asymptomatic incubation period, have you had sex
with any yellow people recently? There are 5 types of hepatitis with 2 transmission methods and 1 presentation.
* Diagnosis is via anti-hepatitis A, C, D, E serology, IgM acutely, IgG chronically. Hepatitis B is the only with
surface antibody, surface antigen, core antibody, core antigen, and e antigen.
* If you are positive for surface antigen (sAg), you are infectious.
* Acute or chronic is determined by the surface antigen being present beyond six months. That means you cannot
tell just from the serology. You have to have the time-course with it.
* Core antigen could be positive or negative, it doesn’t determine if HepB is chronic.
* Vaccination against HepB gives you surface antibody only, so serology with just positive sAb is vaccinated.
* Treatment for acute hepatitis is nothing. Chronic HepA is treated with nothing, there is no chronic HepA.
* Chronic HepB is treated with interferon or (or) lamivudine. Lamivudine, an HIV retroviral, gets rid of HepB sAg.
Most common cause of cancer death in the world is hepatocellular carcinoma. Most common in U.S. is lung cancer.
* Chronic HepC is treated with interferon and (and) ribavirin. Interferon alone is 5-15% cure rate, combined with
ribavirin is 40-50% cure rate. HepC is the most common reason for needing a liver transplant in the U.S. For every
organ that becomes available (heart, lung, liver, pancreas) there are 4-5 people waiting.
* It helps memory to think about what this actually means. Lamivudine is an oral pill to prevent the most common
cause of cancer death in the world. Ribavirin/interferon can help prevent the most common cause of liver
transplantation in the United States.
* Chronic HepE does not exist. Chronic HepD is treated the same as HepB because you have to have HepB to get D.
Sexually Transmitted Diseases
* Patient presents saying their penis is burning, has urinary frequency. How do we tell if it is urethritis or cystitis.
Ask if there is discharge. If discharge, this differentiates urethritis. Next step is swab, culture, and treat. Stain, try to
look for Gram-negative diplococci, try to find intracellular Gonorrhea. Same for cervicitis, swab, culture, treat.
* Treatment is ceftriaxone (for gonorrhea) and azithromycin (for chlamydia). Treatment could be ciprofloxacin (for
gonorrhea) and doxycycline (for chlamydia), or any combination, just so you cover both gonorrhea and chlamydia.
* 27yo man comes to clinic with genital ulcer and adenopathy. Decision point is painful versus non-painful.
Presentation seen in syphilis chancre, chancroid, herpes (HSV), and lymphogranuloma venereum (LGV). “-oid”
means similar, something like cancer would be carcinoid, something like leukemia would be leukemoid.
* Syphilis chancre is painless and firm/indurated, chancroid is painful and soft.
* 27yo man has adenopathy, painless and firm ulcer. What is the next step? Is it RPR, VDRL? Only about 75%
sensitivity with VDRL and RPR in primary syphilis because it needs time to make an antibody response. You won’t
find a dark field scope anywhere. However, initial test is a dark field, because it has greater sensitivity for primary
syphilis. Once you get to secondary syphilis, VDRL and RPR are at 100% sensitivity. In tertiary syphilis, you lose

DO NOT DISTRIBUTE - 36 -
Study Notes – Internal Medicine James Lamberg 01Feb2010

some sensitivity because you start to lose antibody over time. The FTA-ABS does not go away, so you do the
RPR/VDRL then FTA.
* You do not need to know dose, but you do need to know route of administration.
* Primary and secondary syphilis, treatment is IM penicillin. Tertiary syphilis, treatment is IV penicillin.
* What is patient is allergic to penicillin? Answer for primary/secondary syphilis is doxycycline. In tertiary syphilis,
doxycycline will not cross the BBB very well, so treatment is desensitize and treat with penicillin.
* Desensitize and treat with penicillin in pregnancy as well.
* In real inflammation, like meningitis, anything will pass through the BBB, even an aminoglycoside.
* Neurosyphilis takes about 10 years to develop, but about 1 year in HIV patients.
* Syphilis is rarely seen in our population.
* Chancroid is painful, H. ducreyi, pleomorphic Gram-negative bacillus. Testing can be done with Gram stain and
culture. Lots of treatments available. Answer for treatment is single dose azithromycin.
* Mycobacterium avium prophylaxis with a single weekly dose of azithromycin.
* Azithromycin is fantastic, covers chancroid, chlamydia, cervicitis, urethritis, MAC, sinusitis, otitis, bronchitis,
staph, strep, hemophilus.
* 27yo man comes to clinic with multiple small painful genital fluid-filled vesicles. What is the next best step in the
management of this patient? Answer is treat herpes simplex with acyclovir, famciclovir, or valacyclovir. You see the
genital vesicles, so you do not need to Tzanck prep. If the vesicles become unroofed, meaning they get rubbed and
the top comes off showing an ulcer, then you do a Tzanck and culture with Gram-stain for H. ducreyi and with
serology for LGV. Tzanck smear involves swab and putting on slide, looking for cellular atypia.
* LGV is big beefed-up matted up lymph nodes, ulcers around them. Do serology. Treat with doxycycline.
* Patient has genital warts (condyloma acuminata). Poxvirus gives small little warts. Molluscum contagiosum gives
warts with an umbilicated center. Diagnostic test for warts not smear, stain, biopsy, swab. Test is visual
identification of wart. Treatment is remove, freeze (cryo), burn, chop, melt, cut, whatever, just remove it. None will
cure the virus anyway. Imiquimod is a local immunostimulant and the only treatment that does not go too far and
burn the skin (e.g. like cryo would). It stimulates the T-cells to react to the wart as foreign and sloughs it off.
Medication Side-Effects
* Interferon major adverse effect is flu-like symptoms. Why do board exams emphasize adverse effects? Say we’re
in 1970, treatment of CHF would be digoxin and diuretic. In 1985 for a short time we used hydralazine and nitrates,
dig, diuretics. We found hydralazine wasn’t as good as ACE inhibitors. So care was ACE-I, diuretics, dig. Now we
know dig is lousy and does not decrease mortality. So now treatment is ACE-I, beta blockers, diuretics. What is the
most common side-effect of digoxin, GI symptoms (nausea, vomiting). Cough with ACE-I. No matter what the
standard of care is, the adverse effects will not change. So if you’re writing board questions and you choose side
effects to ask about, you won’t have to edit questions much in the future.
* Interferon causes aches, pains, irritability, depression, because you make interferon yourself. That is interferon in
your body. You get the same symptoms with the flu.
Cystitis & Pyelonephritis
* 39yo women with urinary burning, frequency, urgency, dysuria. Is this cystitis or pyelonephritis? It is the same for
both. Best initial diagnostic test for both is urinalysis. Most important thing to look for is WBCs. Bacteria in the
urine is only significant in one patient population, pregnant patients. 1/3 of women with bacteria will develop
pyelonephritis; preterm delivery associated with bacterial vaginosis.
* Best diagnostic test for cystitis and pyelonephritis is a culture. You can tell these apart by the pain. Fever can help
a little too because there is a little fever in cystitis and big fever in pyelonephritis. Suprapubic pain in cystitis and
flank pain in pyelonephritis.
* Cystitis treated with TMP-SMX or ciprofloxacin for 3 days. Usually length of treatment is a matter of local tribal
customs. Cystitis is such a routine problem that there is a standard length of therapy, 3 days for uncomplicated. 7
days is for stones, strictures, tumors, pregnant, or other anatomic complication.
* Pyelonephritis is a drug that covers Gram-negatives. That could be aminoglycosides, ampicillin, fluoroquinolones,
aztreonam, piperacillin, ticarcillin, azlocillin. There is no specific drug, just pick what covers Gram-negatives.
* An ultrasound looking for pyelonephritis will show nothing. You use the ultrasound because you know the patient
has pyelonephritis and want to see what the cause is. Is there an anatomic defect (stone, tumor, hydronephrosis)?
* Say you put patient on antibiotics and 3 days later they are still sick. Now what is the next step? Ultrasound or CT
or MRI and you find an abscess around the kidney. The most accurate test for abscess biopsy or aspirate. You can’t
stain a CT, you can’t get sensitivity on an MRI, you can’t Gram stain an ultrasound.
* Do you have to drain the abscess or can you treat with antibiotics? Most of the time you drain abscesses. The
problem is this happened while you were on antibiotics. So, treatment is to drain the abscess.

DO NOT DISTRIBUTE - 37 -
Study Notes – Internal Medicine James Lamberg 01Feb2010

Skin Infections
* Scabies, 5 times smaller than lice, sarcoptes scabeii. He’s digging and pooping. Diagnostic test is scraping. Might
see trails dug under skin in skin creases. There is itching because of the eggs and feces, contact dermatitis.
Treatment is lindane or permethrin. Lice (pediculosis, crabs if genital) is found near hair areas. Treatment is lindane
or permethrin cream.
* What is the most superficial of all the bacterial skin infections involving just the epidermis? Weeping, oozing,
honey-colored crusting of skin because it lifts up the epidermis. Answer is impetigo, strep and sometime staph.
Staph here is staph aureus (not staph epidermidis). Staph epidermidis lives on your skin and therefore does not cause
infections of the skin.
* Which skin infection involves the epidermis and extends into the dermis? Causes swelling in dermal lymphatic
channels and bright-red skin. Answer is erysipelas, caused by strep. The strep in these skin infections is group A
beta-hemolytic strep, strep pyogenes. You can’t get rheumatic fever from this, but you can get glomerulonephritis.
* Which skin infection involves the dermis and subcutaneous tissues? Answer is cellulitis, staph and strep. It does
not get to the dermal-epidermal junction so it cannot cause oozing above skin.
* If you’re not sure what type of skin infection this is, what do you use? Answer is oxacillin, cloxacillin,
dicloxacillin, nafcillin. If you have a mild penicillin allergy then use first-generation cephalosporin. If life-
threatening penicillin allergy, vancomycin or macrolides.
* Impetigo can sometimes be treated with topicals like bacitracin or mupirocin. It is the only one you can treat with
topicals because it is superficial.
Superficial Fungal Infections
* Any form of superficial fungal infections has the best initial test of KOH and culture. Scrape area, put some KOH
on it, some acetic acid, heat it up, then look at the slide. KOH melts away the epithelial cells, leaving fungi. Fungi
has chitin in the cell wall of the fungus, it is the same stuff that makes the lobster/crab shell hard.
* The KOH will show the fungi. The only way to know which specific fungus is to do a culture. Molds (e.g.
athlete’s foot) take weeks for the cultures. Oral thrush, vaginal candidiasis, cryptococcus are yeasts.
* Fungus is the group, inside the group are molds and yeasts. Inside you at body temperature is yeast. Environment
growing in the refrigerator or plants tends to be molds. “Mold in the cold, yeast in the beast.” Only mold to worry
about in general is skin infections, like athlete’s food. Others would be risopus and mucor mycosis in diabetics.
* Molds include epidermophyton, dermatophyton, malassezia furfur, microsporon. Treatment is based on whether or
not there is hair/nail involvement or no hair/nail involvement.
* Mold treatment without hair/nail involvement would be any topical, clotrimazole, lotrimin, miconazole,
ketoconazole, nystatin, terconazole, miconazole, econazole.
* Mold treatment with hair/nail involvement cannot be topicals. Must be systemic therapy, terbinafine. Kid comes
with toenail infection, athlete’s foot spreading, can also give itraconazole. Treatment for 12-weeks, fingers for 6-
weeks, 80-90% cure.
* Griseofulvin had to be used for 12-18 months for nail infections, only worked 30-40% of the time.
* We don’t use ketoconazole usually; most common side-effect is gynecomastia. Ketoconazole also affects the liver
due to P450 interaction. It is very anti-androgenic that systemically you are more likely to use it for metastatic
prostate cancer; it is very close to orchiectomy at high doses.
* Fluconazole is a yeast drug, good for cryptococcus, vaginal candidiasis, oral thrush, yeast/candida in blood. It does
not work on molds. Fluconazole is very safe.
Osteomyelitis
* 57yo male diabetic with peripheral vascular disease and has pain in leg for last 10 days. It hurts just below his
knee, has an ulcer over the area over past 5 days with a draining sinus tract. Rubor, dolor, tumor, calor. What is the
best initial diagnostic test? CT, bone scan, x-ray, MRI, culture? Answer is x-ray, even if the person has only had the
disease for 5 days. Doesn’t it take 2-3 weeks for the bone to dematerialize enough to see it on an x-ray? Yes, but do
the x-ray anyway. You don’t skip an ECG and jump straight to the stress test. If the x-ray shows osteomyelitis,
periosteal elevation, destroyed bone involucrum and sequestrum.
* X-ray shows osteomyelitis. What is the next step? Answer is biopsy, then treatment. So get the culture then initiate
treatment. How do you know what organism to treat? Most common cause of osteomyelitis is staph aureus.
However, diabetics are more susceptible to Gram-negatives and anaerobes. So you do not know what organism to
treat, which is why you do the biopsy.
* When do you get a SED rate? You get it to follow the response to therapy. You start the patient on 6-weeks of
antibiotics after the biopsy, then after 6-7 weeks or so you check a SED rate. You follow the ESR. If the SED rate is
down then you can stop, if the SED rate is still up then you continue.
* What if the original x-ray were negative, then what is the next step? Answer is MRI. Do not do a bone scan

DO NOT DISTRIBUTE - 38 -
Study Notes – Internal Medicine James Lamberg 01Feb2010

because the red ulceration soft tissue swelling gives too many false positives. MRI has same sensitivity as bone scan
but does not come with the false positives. Cellulitis with red draining tissue will mess up the bone scan. If MRI or
bone scan is positive, next step is biopsy to determine organism.
* If x-ray and MRI negative, then this is just an ulcer, not osteomyelitis.
* When do you culture the drainage out of the sinus tract? Answer is never. It will be contaminated and colonized
with skin flora. It will not tell you what is in the bone.
* Do not culture sinus draining tracts due to false positives. The most accurate test for any infection is culture (few
exceptions, like herpes encephalitis with PCR). X-ray is always first with osteomyelitis even if you think it will be
negative. Never wait for results of culture to treat. SED rate is useful to determine length of therapy. Some people
may need 4-6 months of therapy; this way you do not need to do a bone biopsy every few months, use ESR.
Septic Arthritis
* 72yo female Catholic nun from Brazil with a fever. She has a painful, swollen, red, hot, tender, immobile, effusive
knee. What is the best initial diagnostic test. Answer is aspiration. X-ray very rarely shows anything in septic
arthritis. X-rays, even in rheumatoid arthritis, have a 6-month lag time. Articulatory cartilage is amazing stuff, takes
huge amount of stress and trauma, bacteria cannot penetrate, and considering it is 97% water. So again, do not order
scans here, do an aspiration; tap the knee, arthrocentesis.
* Say arthrocentesis shows 62,000 white cells, polys. What is the most accurate diagnostic test for this patient?
Answer is culture. When do you wait for results of culture? Never. Take culture and start to treat. What type of
septic arthritis does this patient have? You have to order tests, do a Gram stain.
* Non-gonococcal septic arthritis, most often staph (40%) and could be strep (30%), 20% gram negative rods, 10%
misc. What is the sensitivity of a Gram stain? At least 50-70%. Synovial lining does not have a basement membrane,
so bacteria (and antibiotics) pass easily. Culture of synovial fluid is over 90% sensitive. Blood cultures positive in
25-30%. So for non-gonococcal septic arthritis, we’re going to find it.
* Treatment is with two agents to cover Gram-positive and Gram-negatives. Gram-positives with something like
oxacillin, nafcillin. Gram-negatives with something like third-generation cephalosporin (ceftriaxone), quinolones.
There is not specific drug to learn (like ceftriaxone in meningitis), just cover bacteria present.
* What about gonococcal septic arthritis? Blood positive in less than 10%, Gram positive in less than 25%, culture
positive in only about 50%. Treatment is ceftriaxone. How do we make the diagnosis then? Rectum positive in 10-
20% of patents, urethra positive in 10-20% of patients, pharynx positive in 10-20%, cervix positive in 20-30% of
patients. So, if you think there is gonorrhea, culture everything, all these areas. What is going to tell you to do all
these cultures? Is there a rash (non-blanching petechial rash) that goes along with neisseria infections? Migratory
polyarteritis, polyarthralgia (in multiple locations)? Tenosynovitis? That is how you know.
* Many physicians will not even take a sexual history because they assume it is unreliable. Are you sexually active?
No, I just lay there.
Endocarditis
* 27yo man, IVU (IV drug use), has fever, and murmur. You won’t get asked diagnosis. You look in this guy’s eyes
and see no Roth spots. You look in hands and feet and see no Osler nodes. You look in this guy’s fingers and see no
splinter hemorrhages. No Janeway lesions. Urine has no red cells. What does that mean? Nothing. Less than 10% of
patients have any of these signs. All you need to see is fever and a new murmur.
* Dr. Osler invented residencies in this country and wrote the first comprehensive medical text, 700 pages, only 40%
of disease had treatments. Only single disease with 100% mortality was endocarditis. In 19th century, this is how we
think about metastatic disease. These patients dies of post-infectious endocarditis. Now the most common cause of
death from endocarditis is CHF. Osler lived 50 years before antibiotics were invented.
* What is the next best step in the management of this patient? Blood cultures or echocardiogram? Answer is blood
cultures. If blood cultures are positive, then get transthoracic echo. If TTE is negative, then get transesophageal echo
(TEE). If TTE is positive, then you know this is endocarditis.
* Sensitivity of TTE is 50-60% sensitive. That is why you order blood cultures first. TTE will miss almost half of
endocarditis. TEE is more than 90% sensitive, but that is pretty invasive.
* Treatment is started right away, even after blood cultures. When do you answer surgery? What is the most
important and urgent indication for surgery in endocarditis? Answer is CHF. No matter how long you give
antibiotics, the papillary muscle and chordae tendineae will not jump out and reattach themselves.
* To get endocarditis, you need a bacteremia causing procedure (e.g. IV drug use, hemicolectomy, biliary surgery,
dental cleaning) and a significant defect (e.g. aortic stenosis, mitral stenosis, VSD).
* Prophylaxis for general procedures like dental work is amoxicillin 2grams one hour prior. No dose after the
procedure. If patient is penicillin allergic, give clindamycin, azithromycin is alright. Erythromycin will cover the
mouth flora, but has side-effects (vomiting, increased motilin), so not used primarily.

DO NOT DISTRIBUTE - 39 -
Study Notes – Internal Medicine James Lamberg 01Feb2010

* Summary: you see fever and murmur, get the blood culture. If negative, get echo and you’re done.
* You’re going to get your teeth cleaned and have MVP, or pacer in place, CABG, aortic stenosis, mitral stenosis.
So you have some heart problem and are getting a bacterial flinging surgery, cystoscopy, hemicolectomy, whatever.
Now what? MVP, give prophylaxis if significant regurgitation (murmur). Pacemaker, no prophylaxis needed, even
though it is a foreign body. CABG, no prophylaxis needed, coronary arteries are on the outside of the heart.
Prophylaxis for endocarditis when something is on the inside. C-section (with valve problem), no prophylaxis,
uterus is sterile environment. Cardiac catheterization (with valve problem), no prophylaxis, does not cause
bacteremia. IHSS (HOCM) idiopathic hypertrophic subaortic stenosis, needs prophylaxis because it is significant
enough to cause sudden death.
Lyme Disease
* A bunch of children are brought to the hospital with rash, fever, and joint pain. The physicians do tests, ANA
negative, rheumatoid factor negative, SED rate a little elevated (and SED rate is like Rorschach inkblot test, it
doesn’t mean anything). Physicians send kids home and say children have juvenile rheumatoid arthritis. Moms look
into their pediatrics textbook of medicine and decide to stop back to the doctor’s office the next day. All the children
in an area won’t get JRA at the same time. This is how Lyme disease was described, and that is the problem with
diagnosis today. It has a high incidence with the wealthy population. Lyme in Connecticut (near Yale, 3rd highest
income per capita in world). Second highest incidence of Lyme in Westchester NY which has the 2nd highest per
capita income in the world.
* There is no malaria vaccine. There is no chlamydia trachomatis vaccine, 3-30 million people go blind annually
because they don’t have a dollars worth of doxycycline or erythromycin. Lyme, 15000 cases per year and it is very
hard to die from. You can get joint problems, cardiac problems, possibly a few AV block cases, and some
neurologic problems (e.g. Bell’s palsy). Mortality on Lyme is negligible, but can get nasty arthralgia later on. But,
there is a Lyme vaccine. 15,000 cases per year and practically no one dies.
* The fascination from Lyme comes from parents coming back and back and back until the disease was described.
You can get just the rash, or just the neuro, or just any of them. Characteristic about CN VII palsy is that it is
bilateral Bell palsy.
* Tick ixodes scapularis has to be attached for 24-72 hours.
* Diagnosis is clinical manifestation. Serology for IgM and IgG is lousy because it cannot distinguish well between
old and current infection. If you have a positive serologic test and no symptoms, you do not have Lyme. Positive
blood test with no symptoms means no Lyme; this is not like syphilis with VDRL and RPR (diagnosed with syphilis
even if no symptoms). Without clinical manifestations, serology is useless for Lyme.
* Treatment is doxycycline or amoxicillin, for minor stuff like rash or Bell palsy or joint problem. If cardiac or
neurologic problems, use ceftriaxone.
* 26yo woman who gets bitten by a tick on her buttocks. She has no symptoms. What do you do for this patient?
Answer is tickectomy, remove the tick. Then what? Do nothing. No need to treat. No need for tick analysis. No need
for serology (positive serology wouldn’t matter). You do not treat asymptomatic serology.
Human Immunodeficiency Virus (HIV)
* Asymptomatic patient with HIV presents to your office asking what medications they should be taking. Is
prophylaxis based on the T-cell count or the viral load? Answer is T-cell count only. Prophylaxis is based on what
you are at risk of today; T-cells tell the present. Viral load tells the future, how fast will T-cells drop, how aggressive
the diseases is, and when to start anti-retrovirals, HIV drugs, protease inhibitors, reverse transcriptase inhibitors. It’s
like a train heading toward a cliff. The T-cells tell you how far the train is away from the cliff and the viral load tells
you how fast the train is moving toward the cliff.
* Less than 200 T-cells and less than 50 T-cells is what you need to know. Normal is around 600-1000 T-cells. You
don’t feel 400 or even 350. Worry when you get near 200.
* 200 T-cell or less, at risk for pneumocystis pneumonia (PCP), prophylaxis with TMP-SMX. Patient may get a rash
sometimes. If you get a rash, use dapsone. Third line is atovaquone. Aerosol pentamidine is 4th line and practically
an extinct drug.
* 50 T-cell or less, at risk for mycobacterium avium intracellular (MAI, MAC), prophylaxis with azithromycin.
Rifabutin is a dead drug.
* No mortality benefit with antifungal prophylaxis with fluconazole. CMV with oral ganciclovir also not used.
* Asymptomatic HIV-positive man comes to your office with 12 lonely T-cells, what should he be started on?
Answer is TMP-SMX and azithromycin?
* What vaccinations should an HIV positive person be on? Answer is influenza annually and pneumococcal.
* When to start HIV medications, based on T-cell count of viral load? CD4 under 350 or viral load over 55000.

DO NOT DISTRIBUTE - 40 -
Study Notes – Internal Medicine James Lamberg 01Feb2010

* Zidovudine (AZT) causes anemia. Didanosine (DDI), stavudine (D4T), zalcitabine (DDC). Side-effects of DDI
and D4T are neuropathy and pancreatitis. Lamivudine (3TC) has such few side-effects that the placebo has more
side-effects during testing. Reverse transcriptase inhibitors are -vudine drugs. With these 5 medications, patients
were taking pills all the time but still dying anyway.
* Protease inhibitors were created de-novo, not by screening thousands of compounds. These are very effective.
They kill a virus so we call them -avir. Indinavir, saquinavir, nelfinavir, ritonavir.
* Use 2 nucleosides and a protease inhibitor (PI) and viral load goes to undetectable in 80-90% of patients. This
means you cannot die from the virus.
* About 5% of patients who are HIV positive will never develop AIDS. What determines what length of time you
develop AIDS, meaning T-cells from > 500 down to < 200? Answer is viral load. The rate of decay of T-cells is
dependent on the viral load. 95-98% of patients who are HIV positive without treatment will die from AIDS.
* So protease inhibitors help drop viral load to undetectable in 80-90%. So how much has the death rate from HIV
dropped in the United States? Answer is 80-90%.
* Protease inhibitors have side-effects of hyperglycemia and hyperlipidemia. You’d rather be alive with a high LDL
and HbA1c then dead with normal levels.
* Put patient on 2+1 therapy, they’re on it for 3-months, comes back with terrible flank pain and hematuria. Which
drug caused this? Answer is indinavir, causes stones.
* Once you start HIV medications and you go undetectable, how long do you continue the medications? If you keep
patient on medications for 5 years, they are undetectable, stop medications, the virus can come back the next day.
So, patient is on medications forever.
Post-Exposure Prophylaxis
* 26yo surgical intern is stuck with an HIV positive needle (needlestick) in the O.R. by her attending physician.
What is the next best step in management. Answer is give 2 nucleosides and 1 protease inhibitor for one month.
* 33yo man has unprotected sex with an HIV positive girl, who tells him afterwards. What is the next best step in
the morning? Answer is give 2+1 for one month.
* 27yo woman is sexually assaulted by an unknown man with unknown HIV status? What medications do you give
her? Answer is 2+1 for one month.
HIV Transmission Risk
* 32yo woman who is pregnant and is HIV positive. She is at 7 weeks of pregnancy. Her T-cell count is 35, viral
load of 600,000. What do you do for this patient? Answer is 2+1 now. You start HIV medication for < 350 T-cell or
viral load > 55,000. Do not let her die because she is pregnant. This woman has a 50% six month mortality.
* Which HIV medications (nucleosides or PI) are teratogenic? None of them.
* Pediatric HIV is virtually non-existent in the U.S., less than 400 children born HIV positive. Average life
expectance of an HIV positive person who gets the disease around the age of 40 is the same as an HIV negative
person provided they get the 2+1 therapy.
* AZT alone in pregnancy results in about an 8% transmission of HIV. With 2+1 therapy, transmission rate drops
even further to about 2%.
* In the United States, average life expectancy for a woman is 79 years. In South Africa, it is 32 years, mainly due to
the fact that 1/3 of women in prenatal clinics are infected with HIV. Half of those children will be infected and most
of those children will be dead by age 5. Have gratitude in your attitude for access to the medical therapy we have.
* Risk of transmission is about 1:3000 for female to male (vaginal). Risk is about 1:1000 for male to female
(vaginal). Needlestick exposure is about 1:300 episodes/contact. Anal receptive is about 1:100 (1%) per shot, load,
event. Mother to child transmission is about 1:20 with meds and about 1:3 or 1:4 without medications. C-section not
routinely indicated, only if viral load is not under control. Risk of transmission through breast milk is equal to
unprotected sex or equal to the risk from the delivery itself. In Africa, death rate is higher since there is no milk
formula available. Oral transmission rates are not really known, it is a difficult thing to study.
* Why is male to female transmission higher than female to male? Answer is not vaginal trauma. Answer is not
surface area differences. Answer is duration of contact with infected semen for 4-5 hours. Man is only exposed for
about, what, 10 seconds? The time duration of the coitus, less than 4-5 hours.
* Over 70 years, there are about 5 drugs created for tuberculosis. Over 10 years, 16 HIV medications were approved
and new ones coming out all the time. You took a fatal prognosis with HIV, and made it 80-90% not fatal. You took
a transmission risk of 50% and brought it down to 2-3%, and that is a great good.
Medical Examination Side-Note From Dr. Conrad Fischer
* You’re sitting there looking up at that board exam and all you see is a huge exam. You feel scared and all you can
think about is how big that exam is. But, if you could find a way to look past the exam and see good and beauty.
You could connect with the goodness and beauty that is beyond the exam, which you can’t do right now because all

DO NOT DISTRIBUTE - 41 -
Study Notes – Internal Medicine James Lamberg 01Feb2010

you see is a giant exam. But, if you can connect with the knowledge, the data that you need for the exam and
connect it with the beauty and good, great things will happen. First, the exam will seem much smaller and lighter in
the palm of your hands (e.g. William Blake). Second, you will be about to remember the knowledge longer and help
someone later on with it, and that is a great good. Third, you will get a better grade anyway and get what you want.
If doing well on the exam is your highest aspiration, it will become so painful. Fourth, your trip/voyage/journey will
be filled with much more joy. This idea is the analgesic for your studying.

Kaplan Videos – Dermatology with Dr. NAME, MD


To Be Completed

Kaplan Videos – Gastroenterology with Dr. NAME, MD


To Be Completed

Kaplan Videos – Endocrinology with Dr. NAME, MD


To Be Completed

Kaplan Videos – Hematology with Dr. NAME, MD


To Be Completed

Kaplan Videos – Cardiology with Dr. NAME, MD


To Be Completed

Kaplan Videos – Rheumatology with Dr. NAME, MD


To Be Completed

DO NOT DISTRIBUTE - 42 -

You might also like